You are on page 1of 181

#WINNER STROKE COLLECTION#

Directions (1-5): Study the following information carefully and answer the
questions given below. A Guest house accommodates twelve persons in six
rooms. The persons are A, B, C, D, E, F, P, Q, R, S, T and U. The rooms are
1, 2, 3, 4, 5 and 6. Each room of the guest house is coloured with different
colours. Those colours are blue, green, yellow, black, pink and white. Each
room accommodates two persons.
A, B, Q, R, T and U are the only males in the group. Female’s rooms are odd
– numbered whereas male’s rooms are even-numbered.

i. A and B share room. C’s roommate is not D.


ii. E does not live in 6, which is yellow. Neither room 5 nor room 4 is blue or
black.
iii. The pink room is an odd number but it is not room 3. F lives in room 5
with D
iv. P’s roommate is not S. The blue room is even numbered.
v. R lives in the green room whereas D lives in the white room.
vi. S does not lives in room 3. T’s room is blue.

1.Who among the following are accommodated in the pink-coloured


room?
1 : E and S
2 : C and S
3 : P and C
4 : Data inadequate
5 : None of these

ANS-4

2.Who among the following are accommodated in room 3?


1 : P and C
2 : P and E
3 : C and E
4 : Data inadequate
5 : None of the above
ANS-4

3.In which A and B are accommodated?


1:2
2:4
3:6
4 : Data inadequate
5 : None of these

ANS-3

4.Which of the following statements are definitely true?


i. U is the partner of T
ii. Q is the partner of D
iii. R is the partner of Q
iv. U is the partner of R
1 : Only i
2 : Only ii
3 : Only iii
4 : Both i and ii
5 : None

ANS-5

5.Which of the following statements is/are definitely false?


(A) A and B are accommodated in the yellow room
(B) Q is accommodated either in the blue room or in the green room
(C) C is accommodated either in the Pink room or in the black room
(D) U is accommodated in room 2
1 : None
2 : Only D
3 : Only A & D
4 : Only B & D
5 : All are true

ANS-2
Directions (6-10): Study the following information carefully and answer the
questions given below.
A dealer sold six bikes – Avenger, Bullet, Champ, Desire, Enfield and Fazer –
during a period of Monday to Saturday, one bike on each day.
i. The bike Champ was sold at least before three bike
ii. The bike Fazer was sold on Tuesday
iii. Both the bike Bullet and Enfield were sold at least before one bike.
iv. The bike Desire was sold immediately after the bike Champ.
v. At least four bike were sold after the bike Enfield.

1.How many bikes were sold after the bike Desire?


1 : Can’t say
2:3
3:2
4:5
5 : None of these

ANS-3

2.How many bikes were sold before the bike Fazer?


1 : Can’t say
2:1
3:2
4:3
5 : None of these

ANS-2

3.The bike Avenger was sold immediately after:


1 : Monday
2 : Tuesday
3 : Wednesday
4 : Friday
5 : Saturday
ANS-4

4.If Ravi is the person who purchased the bike sold on Wednesday,
then the bike purchased by him is:
1 : Avenger
2 : Bullet
3 : Champ
4 : Desire
5 : Enfield

ANS-3

5.Which is the bike sold on Saturday?


1 : Bullet
2 : Can’t say
3 : Champ
4 : Desire
5 : None of these

ANS-5

Four persons sitting in a row-I, P, Q, R and S are facing South and the four
more persons sitting in another row-II T, U, V and W are facing North. Each
of them is from a different country. One of those countries is Australia. The
following is known about them.

1.The person from England is sitting second to the left of S and W is adjacent
to the person who faces the person from England.
2.Exactly one person sits between W and the person from Pakistan, who is
not V.
3.The person who is second to the right of the person from Pakistan faces
the person from India.
4.The person from U.S.A faces the person from Japan. S is neither from
Japan nor adjacent to the person from India.
5.R faces the person from Bangladesh who is not W. The person from China
is not facing the person from India.
6.P is not at an extreme end and is not facing T. V is not from Bangladesh.

1.How many persons are sitting between S and Q?


1 : None
2 : One
3 : Two
4 : Either (one) or (two)
5 : Cannot be determined

Answer

ANS-3 :Two

2.Who are the persons sitting at extreme ends?


1 : S and the person from Pakistan.
2 : Q and the person from Bangladesh.
3 : W and the person from England.
4 : V and the person from USA.
5 : Data inadequate.
Answer
4 :V and the person from USA.

3.In the given arrangement in a certain way V is related to P and U is


related to R in the same way. Who is related to T?
1:P
2:W
3:S
4:Q
5 :Data inadequate
Answer
4 :Q

4.Find the odd one out.


1:S
2 : The person from India
3:W
4 : The person from Japan
5:P
Answer
5:P

5.If U is from the USA and T is from China, then W is from______.


1 : England
2 : Pakistan
3 : India
4 : Japan
5 : U.S.A
Answer
1 :England
Directions for questions : These questions are based on the following
information.
Eight persons A, C, E, F, G, L, M and P have different sim cards, viz. Docomo,
Airtel and BSNL. There are atleast two persons at most three persons who
have the same sim. They have recharged their sim with different amounts viz
Rs.20, Rs.30 and Rs. 40. No two persons, who have the same sim, recharged
with the same amount. Atleast two persons and atmost three persons
reacharged with the same amount.
P recharged with Rs.40 and has BSNL sim. F and P have the same sim but F
did not recharge with Rs.20. F and L recharged with the same amount. The
persons with Docomo sim, did not recharge with Rs.30. G and E recharged
with the same amount. G and M have the same sim which is not the sim of L.
A and C recharged with the same amount but not with Rs.40. C and E have
different sims.

1.Who among the following persons has Airtel sim?


1:C
2:M
3:G
4:F
5:E
5:E

2._______sim does A have.


1 : BSNL
2 : Airtel
3 : Docomo
4 : Either Docomo or Airtel
5 : Cannot be determine

2 : Airtel

3.Choose the correct combination.


1 : G – 40 – BSNL
2 : L – 20 – Airtel
3 : F – 30 – BSNL
4 : A – 20 – BSNL
5 : C – 20 – Airtel

3 : F – 30 – BSNL

4.Four of the following five are alike in a certain way based on the
given information and hence form a group. Find the one, which does
not belong to that group?
1 : A, Airtel
2 : C, BSNL
3 : E, Rs.30
4 : M, Rs.20
5 : G, Rs.40

3 : E, Rs.30

Directions for questions: These questions are based on the


information given below.

Eight members – P, Q, R, S, T, U, V, W of a family are sitting around a


circular table. The family members belong to three generations and each
couple has at least one daughter and one son.
Q, the grand father of U, has more number of children than the number of
grandchildren. S is opposite his mother T, who is adjacent to her
granddaughter. V is to the immediate left of her brother-in-law. P’s daughter
R, is to the immediate right of her aunt W. P is third to the right of his sister.

1.What is the position of R with respect to T?


1 : Immediate left
2 : Immediate right
3 : Second to the left
4 : Second to the right
5 : Either (1) or (2)

1 : Immediate left

2.How many couples are there in the family?


1 : One
2 : Two
3 : Three
4 : Four
5 : Cannot be determined
2 : Two

3.What is the position of Q with respect to S?


1 : Second to the left
2 : Immediate right
3 : Second to the right
4 : Immediate left
5 : Cannot be determined

5 : Cannot be determined

4.How is S related to U?
1 : Uncle
2 : Mother
3 : Father
4 : Aunt
5 : Grand mother

1 : Uncle

5.Which of the following pairs are not siblings?


1 : SW
2 : PS
3 : UR
4 : WP
5 : PV

5 : PV

Directions for questions : These questions are based on the following


information.

Each of the six persons A, B, C, D, E and F belongs to a different tribe among


P, Q, R, S, T and U. Each of them belongs to one of the four countries
Bhutan, China, Mangolia and Pakistan. At least one and at most two persons
belong to each of the four countries. Each of them knows exactly one
language among Spanish, French, Dutch and Swedish. At least one and at
most two know each of these four languages. No two persons have the same
combination of country and language. The following information is known
about them.

(i) F and C belong to the same country and A and D belong to different
countries.

(ii) E belongs to China and tribe T. The person who belongs to tribe P knows
Swedish.

(iii) Two persons know Dutch and two persons belong to each of Bhutan and
Pakistan.

(iv) B belongs to Bhutan. Two persons the one who belongs to the tribe R
and B know French.

(v) C knows Dutch and belongs to tribe Q.

(vi) Neither A nor the person who belongs to Pakistan knows French.

(vii) The persons who are belonging to the tribes Q and U know the same
language.

1.A belong to ______ country.


1 : Bhutan
2 : China
3 : Mangolia
4 : Pakistan
5 : Cannot be determined.

1 : Bhutan

2.F belongs to ___ Trade.


1:P
2:R
3:Q
4:T
5:S

1:P

3.Choose the true statement about the persons who belong to the
tribes S and U?
1 : One of them is F and the other one is B.
2 : They both know the same language.
3 : They both belong to the same country.
4 : One of them knows Spanish
5 : They do not have any common parameter.

3 : They both belong to the same country.

4.D know ______ language.


1 : Spanish
2 : Dutch
3 : Cannot be determined
4 : French
5 : Swedish

4 : French

5.Choose the true statement about the person who belongs to the
tribe P.
1 : The person belongs to China.
2 : The person is A.
3 : The person knows the same language which the person from the country
Bhutan knows.
4 : The person is from Pakistan.
5 : None of these

4 : The person is from Pakistan.


Directions for questions : These questions are based on the following
information.

Six girls – A, B, C, D, E and F are sitting in a row facing North and six boys –
P, Q, R, S, T and U are sitting in a different row facing South, but not
necessarily in the same order. Each girl in a row is facing exactly one boy
from the other row. The following information is known about them.
P sits second to the right of the person who sits opposite B. B does not sit at
any end of the row. C and D are immediate neighbours of B, and one of them
sits at an extremes. E sits second to the left of D but not opposite R.
P is a neighbour of R but is not opposite F. Neither Q nor T are adjacent to P.
T is adjacent to either R or Q but not both. U is a neighbour of the person
who sits opposite E.

1.________sits opposite to C.
1:S
2:P
3:R
4:Q
5:T

5:T

2._____sits second to the left of R?


1:S
2:T
3:P
4:Q
5:U

2:T

3.If S and D interchange their positions, then who sits to the


immediate left of D?
1:U
2:R
3:P
4:T
5:Q

3:P

4.Four of the following five are alike in a certain way and so form a
group. Find the one which does not belong to that group?
1 : U, A
2 : P, B
3 : D, S
4 : C, R
5 : D, T

5 : D, T

5.Which of the following is true, according to the given information?


1 : U and C are at the extremes.
2 : R sits second to the right of S.
3 : T sits to the immediate right of Q.
4 : B sits second to the left of A.
5 : All the above

1 : U and C are at the extremes.

Directions for questions : These questions are based on the following


information.
Eight persons A, B, C, D, E, F, G and H are sitting around a circular table
facing the center. Each of them works in a different city among Delhi,
Kolkata, Kanpur, Goa, Banglore, Hyderabad, Chennai and Pune, but not
necessarily in the same order.
(1) A is third to the left of the person who works in Goa and opposite G.
(2) B is second to the right of E, who is not to the immediate right of G. E is
not adjacent to A.
(3) The persons who works in Kanpur and Delhi are opposite each other but
they are neither B nor A.
(4) D, who works in Hyderabad, is third to the right of the person who works
in Kanpur.
(5) F and H are neither opposite nor adjacent to each other.
(6) C is neither adjacent to F nor adjacent to the person who works in
Banglore.
(7) Neither A nor G is from Chennai and the person who works in Chennai is
to the immediate left of the person who works in Kolkata.

1.________works in Pune
1:H
2:G
3:A
4:B
5:C

4:B
2.In ______city does A work.
1 : Pune
2 : Kanpur
3 : Banglore
4 : Delhi
5 : Kolkata

5 : Kolkata

3.Who is second to the left of the person from Hyderabad?


1 : The person from Kolkata.
2 : The person from Chennai.
3:C
4:G
5 : None of these

2 : The person from Chennai.

4.Who is seated opposite C?


1:F
2:E
3:D
4:H
5:G

1:F

5.Which of the following combinations is correct?


1 : G-Pune
2 : B-Chennai
3 : F-Delhi
4 : H-Kanpur
5 : None of these

3 : F-Delhi
Directions for questions : These questions are based on the following
information.

A,B,C,D,E,F,G,H, eight members of a family sit around a circular table. The


following information is known about them: There are three married couples
in the family. E is third to the right of B, and exactly two persons sit between
G and E. C is to the right of his grandson. F is the son-in-law of D and his
daughter is A who sits to the left of her aunt E, who is opposite D. G and E
are sisters-in-laws. H is unmarried, while F has no siblings. Each couple has
at least one child.

1.________sits to the right of H.


1:A
2:B
3:C
4:D
5 : None of these

3:C

2.Who is wife of B?
1:C
2:E
3:A
4:D
5:G

2:E

3.______is the daughter of D


1:B
2:A
3:G
4:E
5:F

3:G
4.______sits opposite C’s son.
1:H
2:E
3:G
4:F
5 : Cannot be determined

1:H

5._____is 3 places away to the left of B


1 : His son
2 : His father
3 : His mother
4 : Her father
5 : Either her mother or father

2 : His father

Four persons A, B, C and D are sitting in row-I, facing South and another
four persons – W, X, Y and Z are sitting in row-II, facing North. Each person
in row-I is facing exactly one person from row-II. One of the persons is from
Biotechnology.
B sits second to the left of the person, who is opposite the person from MCA.
The person from MCA is an immediate neighbour of Y. Y is not from CSE, but
is immediate neighbour of the person from IT. W is not from IT, but sits
opposite the person from MBA, who is not at any extreme. A sits opposite the
person from Civil. C is from ECE and is an immediate neighbour of the person
from Mechanical. Z sits opposite the person who is immediate neighbour of
C.

1. _______persons are sitting between D and the person from


Mechanical?
1 : One
2 : Zero
3 : Two
4 : Either (1) or (2)
5 : Cannot be determined

Answer

1 : One

2. ______sits opposite the person from Biotechnology.


1:D
2 : The person from CSE
3 : The person from ECE
4:B
5:Z

Answer

3 : The person from ECE

3. ______sits second to the right of X.


1:Y
2:W
3:Z
4 : The person from MCA
5 : The person from IT

Answer

1:Y

 Four out of the following five are alike in a certain way based on the
given information and so form a group. Find the one which does not belong
to that group?
1:B
2:C
3 : The person from Civil
4:Z
5 : The person from CSE
Answer.2 : C
 Which of the following is true ?
1 : The person from Mechanical is at the right extreme.
2 : C is not sitting opposite Y.
3 : The person from MBA and A are neighbours.
4 : D is from MCA.
5 : ALL of these
Answer
3 : The person from MBA and A are neighbours.

Directions for questions : These questions are based on the following


information.
Of the five boys A, B, C, D, and E, two are good, one is poor and two are
average in studies. Two of them study in post-graduate classes and three in
under-graduate classes. One comes from a rich family, two from middle class
families and two from poor families. One of them is interested in music, two
in acting and one in sports. Of those studying in under-graduate classes, two
are average and one is poor in studies. Of the two boys interested in acting,
one is a post-graduate student. The one interested in music comes from a
middle class family. Both the boys interested in acting are not industrious.
The two boys coming from middle class families are average in studies and
one of them is interested in acting. The boy interested in sports comes from
a poor family, while the one interested in music is industrious. E is
industrious, good in studies, comes from a poor family and is not interested
in acting, music or sports. C is poor in studies in spite of being industrious. A
comes from a rich family and is not industrious but good in studies. B is
industrious and comes from a middle class family.

1. Name the boy interested in sports.


A. A
B. B
C. C
D. D

Answer
C. C

2. Name the boy interested in music.


A. A
B. B
C. C
D. D

Answer

B. B

3. Name the middle class family boy interested in acting.


A. A
B. B
C. C
D. D

Answer

D. D

4. Name the boys studying in post-graduate classes.


A. A, D
B. A, E
C. B, C
D. D, E

Answer

B. A, E

5. Name the boy who is not industrious and is average in


studies.
A. A
B. B
C. C
D. D ANS-D
Directions for questions : These questions are based on the following
information.
There are seven students – A, B, C, D, E, F and G. Each of them has a
different favorite subject – English, Math, Physics, Chemistry, Biology,
Sociology and Philosophy. Each of the students secured different marks in
the examination. C has secured second highest marks and neither English
nor Math is his favorite subject. E secured the least marks and neither
Sociology nor Math is his favorite subject. The favorite subject of D is Physics
and he secured more marks than B and G but less than A. The favorite
subject of B is Biology and his marks are more than the marks of E but less
than the marks of G. the favorite subject of A is Philosophy. The student
whose favorite subject is Chemistry got the highest marks.

1. Chemistry is the favorite subject of


A. A
B. C
C. F
D. G

Answer

C. F

2. Which is the favorite subject of E?


A. Math
B. English
C. Biology
D. Data inadequate

Answer

B. English

3. When all the seven students are arranged according to their


marks in descending order, what is the position of A from the
top?
A. Second
B. Third
C. Fourth
D. Data inadequate

Answer

B. Third

4. What is the favorite subject of G?


A. Math
B. Sociology
C. Either Math or Sociology
D. None of these

Answer

A. Math

5. Who got the second least marks?


A. C
B. D
C. G
D. None of these

Answer

D. None of these

Directions for questions : These questions are based on the following


information.
Banking Complex is an 8 storeyed building located in suburbs of Mumbai. The
seven major banks of the country have their corporate offices on the
different floors of the building starting from 1st floor to 7th floor. 0th or
Ground floor has a parking lot. The different banks are namely CICIC bank,
DFHC bank, Cyndicate bank, VTI bank, Kanara bank, Korporation bank and
HFBC bank. Each of these bank is managed by a manager. The names of the
managers of these banks are Mr. K.Bangarappa, Mr. V.Kant, Mr. S.Raju, Mr.
R.Sanjay, Mr. A.Subodh, Mr. D.Shishir and Mr. B.Krishna (not necessarily in
the same order). Further, information is given as
a. Either VTI bank or HFBC bank is on the top or first floor respectively.
b. D. Shishir is the manager of the bank which is on 5th floor.
c. DFHC and Korporation bank are on the consecutive floors.
d. K. Bangarappa and Mr. A. Subodh are the managers of the CICIC bank
and Kanara bank respectively.
e. Mr. S. Raju and Mr. D. Shishir are the managers of the banks which are on
consecutive floors.
f. Mr. V.Kant, who is the manager of DFHC bank, and Mr. K. Bangarappa
have one bank in between their respective floors.
g. Mr. S.Raju, who is the manager of the Cyndicate bank sits on the floor just
above the Korporation bank.

1. Which bank is on the immediate next floor to Kanara bank?


A. DFHC bank
B. CICIC bank
C. Either (A) or (B)
D. Cyndicate bank

Answer

C. Either (A) or (B)

2. Mr. D.Shishir is the manager of which bank?hich bank is on


the immediate next floor to Kanara bank?
A. Korporation bank
B. Kanara bank
C. DFHC bank
D. Either (A) or (B)

Answer

A. Korporation bank

3. If Mr. R.Sanjay is the manager of the bank on 1st floor, who


is the manager of VTI bank?
A. Mr. R. Sanjay
B. Mr. K. Bangarappa
C. Mr. A. Subodh
D. Cannot be determined

Answer

D. Cannot be determined

4. The manager of the Cyndicate bank sits on which floor?


A. Second
B. Third
C. Sixth
D. Fourth

Answer

C. Sixth

5. If HFBC bank is on top floor, how many banks are in between


DFHC bank and HFBC bank?
A. Two
B. Four
C. One
D. Three

Answer

A. Two

Directions for questions : These questions are based on the following


information.
Amit, Bunty, Reena, Veena and Dolly were selected for prizes because of
their excellent performance by the school. On this occasion their parents
were also invited by the school authority. Brajest, Jayant, Manoj, Ritesh and
Vinod were the fathers of the individuals but not in the same order. Anju,
Babita, Punam, Rita and Vinita were the mothers of the individuals but not in
the same order. Among the children only two are males. The first letter of
the name of any parent was not same as the first letter of eth name of their
child. The same was true of spouses also.
The first letter of the name of Dolly’s father was the same as that of Reena’s
mother. Similarly, Dolly’s mother’s name started with the first letter of
Reena’s father name. Brajesh and Manoj were not the fathers of Veena and
Amit respectively. The first letter of Bunty’s mother was the same as the first
letter of the name of Veena’s father. Anju is not Dolly’s mother. Jayant is the
husband of Punam. Reena’s performance was best among all; that is why her
mother Babita praised her.

1. Who among the following is the father of Amit?


A. Manoj
B. Vinod
C. Jayant
D. Brajesh

Answer

C. Jayant

2. Who among the following is the son of Rita?


A. Amit
B. Bunty
C. Either Amit or Bunty
D. Can’t say

Answer

B. Bunty

3. Father’s name of Reena is?


A. Can’t say
B. Vinod
C. Brajesh
D. Manoj

Answer

B. Vinod

4. Who of the following couples is correctly matched?


A. Vinod, Babita
B. Manoj, Punam
C. Brajesh, Anju
D. None of these

Answer

A. Vinod, Babita

I. Study the following information carefully to answer the given


questions
W, X, Y, Z, M, N and O are travelling in three buses P , Q and R
with at least 2 of them in any of these buses. Each of them has a
favourites country such as Denmark, Russia, Britain, Malaysia,
Germany, Kenya and USA.
X travelling in bus Q with M. M’s favourite country is USA.
Those who travel in bus P do not like Denmark and Malaysia. The one
who likes Russia travels only with O in bus R. The one whose
favourite country is Germany does not travel in the same bus with
either M or O. W does not travel in bus Q. W likes Kenya. Z and N are
travelling in the same bus. N does not like Britain. The one whose
favourite country is Malaysia does not travel in Bus Q.

1. In which bus W, Z and N travelling ?


A)Cannot be determines
B)P
C)Q
D)R
E)None of these

Answer

B)P
Explanation:

2. Whose favourite country is Denmark ?


A)X
B)W
C)Y
D)Z
E)None of these

Answer

A)X

3. What is M’s favourite country?


A)Kenya
B)Malaysia
C)USA
D)Germany
E)None of these

Answer

C)USA

4. Which of the following combination is right ?


A)W – R : Russia
B)O – P : Germany
C)N – R : Malaysia
D)Z – P : Britain
E)None of these

Answer

D)Z – P : Britain

5. Y travelling in which bus ?


A)P
B)Q
C)R
D)Cannot be determined
E)None of these

Answer.C)R
II. Study the following information carefully to answer the given
questions
A group of 7 persons P, Q, R, S, T, U AND V works as Clerk, IT
officer, Agricultural Officer, Forest Officer, Terminal Operator,
Research Analyst and Economist for the banks A, B, C,D , E, F and G.
R works for bank E and is neither a Economist nor a Forest officer. T
is an Research Analyst and works for bank C. P works as a Clerk and
does not work for bank B or D. The one who is an IT Officer works for
bank F. The one who works for bank D works as a Forest Officer. U
works for bank B. V works for bank A as a Economist. S is not an IT
Officer.

1. Who among the following works as Terminal Operator ?


A)P
B)Q
C)R
D)S
E)None of these

Answer

C)R
Explanation:

2. Who working in the bank B ?


A)U
B)V
C)S
D)T
E)None of these

Answer-A)U

3. Which of the following combination is correct ?


A)F – Agricultural Officer
B)C – Clerk
C)A – Economist
D)G – Forest Officer
E)None of these

Answe=A – Economist

4. What is the profession of R ?


A)Clerk
B)Terminal Operator
C)IT officer
D)Economist
E)None of these

Answer -Terminal Operator

5. Which of the following is correct ?


A)S – Bank C
B)V – Bank B
C)R – Bank D
D)P – Bank G
E)None of these

Answer-P – Bank G

I. Study the following information to answer the given questions


questions
P, Q, R, S, T,U and V are seven employess in the Big company.
Three of them are in 1 grade and 2 of them in other 2 grades among
A, B and C. Each of them earns different amount as salary.
There are 3 ladies among them one each in each grade. R and
her husband only are in grade B. V earns the maximum and he along
with only S are in grade A. S earns more than only P. U and her friend
Q are in Grade C. P is not in Grade C. U earns less than R but more
than Q. T earns less than Q.
1. Who are the following in Grade A ?
1)S
2)P
3)V
4)Both S and V

Answer-Both S and V

2. Who earns greater than R ?


1)U
2)V
3)P
4)Both U and V

Answer-V

3. The one who earns the maximum in which Grade ?


1)A
2)B
3)C
4)Data inadequate

Answer-A

4. Who among the following earns the minimum Salary ?


1)V
2)T
3)P
4)R

Answer-P

5. Which of the following Group belongs to Grade C ?


1)PSV
2)TOR
3)QTU
4)QRV

Answer-QTU
II. Study the following information to answer the given questions
questions
7 players A, B, C, D, E, F and G from 7 different countries such
as Canada, India, Singapore, Sri Lanka, South Africa, Kenya and
Maldives participate in the Games but not necessarily in the same
order.
All of them participate in 7 different games, viz Badminton,
Swimming, Boxing, Hockey, Netball, Rugby and Tennis. The one who
participate in Boxing does not belong to South Africa. D is from
Singapore and he does not participate in Hockey. The one who is
from Sri Lanka participates in Badminton. The one who is from India
does not participate in Hockey or Netball. A is from Maldives. G from
Canada and participates in Rugby. C participates in Table tennis but
he is neither from Kenya nor South Africa. B participates in
Swimming and E participates in Boxing.

1. Who among the following Participated in Rugby ?


1)F
2)G
3)D
4)B

Answe-G

2. The one who is from Kenya participated in which of the


following games ?
1)Swimming
2)Hockey
3)Boxing
4)Tennis

Answer-Boxing

3. C belongs to which of the following Country ?


1)India
2) Sri Lanka
3)Maldives
4)Kenya

Answer-India

4. Which of the following Combination is correct ?


1)B- South Africa – Swimming
2)G -India – Badminton
3)D-Maldives – Tennis
4)A-Canada – Swimming

Answer-B- South Africa – Swimming

5. The one who is from Canada participated in which of the


following games ?
1)Boxing
2)Tennis
3)Rugby
4)Netball

Answer -Rugby

Study the given information carefully to answer the given questions.

A, B, C, D, E, F and G live on 7 different floors of building but


not necessarily in the same order. The lowermost floor is numbered
1, then 2,3 and so on. Each one of them also likes a different colours
namely violet, yellow, green, pink, white, blue and red.
A lives on an odd numbered floor but not on the floor
numbered 3. The one who likes white lives immediately above A.
Only 2 persons live between D and the one who likes white.
The one who likes yellow lives on one of the odd numbered
floor above D. Only 3 persons live between C and the one who likes
yellow. The one who likes green lives immediately above C.
The one who likes violet lives immediately above one who
likes red. G lives on an odd numbered floor. Only one person lives
between B and E. B lives on one of the floors above E. Neither C nor A
likes pink. E does not like green.

1. Which of the following colour does E likes ?


A)Red
B)Violet
C)Blue
D)Pink

Answer-Violet

2. Who among the following lives on the floor numbered 5 ?


A)F
B)D
C)B
D)A

Answer-A

3. Which of the following combination is correct ?


A)C – Blue
B)G – Violet
C)A – Green
D)F – Yellow

Answer-C – Blue

4. Which of the following combination is not true ?


A)D – 3th floor
B)G – 7th floor
C)E – 4th floor
D)C – 5th floor

Answer-C – 5th floor

5. Who lives between E and F ?


A)D
B)G
C)C
D)B

Answer-D

II. Study the given information carefully to answer the given


questions.

Seven people P, Q, R, S, T, U and V have their weekly offs on


different days of the week(Sunday – Saturday but not necessarily in
the same order).Each of them has a liking for different cuisine –
Indian, Italian, Mexican, Chinese, Spanish, Thai and Continental not
necessarily in the same order.
Q likes Thai food and get its weekly off on Thursday. S likes
Italian food and does not have off on Sunday. T has weekly off on
Saturday. V has weekly off on Tuesday. U likes Continental food,
where as the one who have weekly off on Monday likes Mexican
food. R does not like Spanish food and has weekly off on Wednesday,
the one who likes Indian food does not have weekly off on Tuesday
or Wednesday.

1. Who likes Indian Cuisine ?


A)R
B)T
C)V
D)S

Answer-T

2. S have weekly of on which of the following Day?


A)Tuesday
B)Saturday
C)Friday
D)Monday

Answer-Friday
3. R like which of the following Cuisine ?
A)Mexican
B)Italy
C)Thai
D)Chinese

Answer-Chinese

4. Which of the following pair is correct ?


A) V Chinese
B) R Spanish
C) P Mexican
D) Q Indian

Answer

C) P Mexican

5. Which of the following pair is not correct ?


A)Q – Thursday
B)S – Friday
C)V – Tuesday
D)T – Monday

Answer

D)T – Monday

I.Study the following information carefully to answer the questions


given below.
A, B, C, D, E, F and G are 7 friends left for 7 different cities –
Delhi, Chennai, Hyderabad, Bangalore, Kolkata, Chandigarh and
Jaipur, each one on a different day of the week. C left for Jaipur on
Monday. On the last day of the week, the person left for Bangalore. E
left the next day of A who left for Chandigarh and on the previous
day of G. D left for Kolkata on Friday. B didn’t leave for either
Hyderabad or Bangalore and G left for Delhi.
1. On which day of the week did B leave ?
A)Sunday
B)Saturday
C)Monday
D)Tueday
E)None of these

Answer

2. Who left for Bangalore ?


A)E
B)B
C)G
D)F
E)None of these

Answer

D)F

3. Who left on Tuesday ?


A)A
B)C
C)G
D)F
E)None of these
Answer

A)A

4. On which day of the week did E leave ?


A)Friday
B)Sunday
C)Wednesday
D)Thursday
E)None of these

Answer

C)Wednesday

5. Which of the following Combination is not right ?


A)C – Jaipur
B)F – Bangalore
C)A – Chandigarh
D)E – Kolkata
E)None of these

Answer

D)E – Kolkata

II. Study the following information carefully to answer the questions


given below.
P, Q, R, S, T, U and V are 7 friends who travel to college
everyday by a particular train which stops at 5 stations – 1, 2, 3, 4
and 5 respectively after it leaves base stations. 3 among them get in
the train at the base station. S gets down at the next station at which
U gets down. Q gets in with 2 persons and does not get down with
either P or T. V alone gets in at station 3 and gets down with R after
1 station. P travels between only 2 stations and gets down at station
5. None of them gets in at station 2. R gets in with U but does not get
in with either Q or S. T gets in with 2 others and gets down alone
after S. Q and S going to same college and they get down together at
station 3. None of them gets down at station 1.

1. At which station does T get down ?


A)Station 2
B)Station 4
C)Station 3
D)Station 5
E)None of these

Answer

2. At which station does R, U get in ?


A)Base station
B)Station 2
C)Station 1
D)Station 3
E)None of these

Answer

C) Station 1

3. After how many station does Q get down ?


A)4
B)3
C)1
D)2
E)None of these
Answer

D)2

4. At which of the following station does Q and T get in ?


A)Base Station
B) Station 1
C) Station 2
D) Station 3
E)None of these

Answer

A)Base Station

5. Which of the following is correct ?


A)T gets in at the base station
B)R gets in at the Station 3
C)V gets down at Station 5
D)U gets down at Station 2
E)None of these

Answer

A)T gets in at the base station

I.Study the following information carefully to answer the questions


given below.
Seven members P, Q, R, S, T, U and V are working in different
cities Ahmedabad, Bangalore, Chennai, Hyderabad, Kolkata, Delhi
and Mumbai not necessarily in the same order. Each one has a
different mother tongue Tamil, Kannada, Telungu, Hindi, Marathi,
Punjabi and Bangla not necessarily in the same order.
R works in Bangalore and his mother tongue is not tamil or
marathi. S’s mother tongue is Punjabi and he works in Ahmedabad. T
and U do not work in Chennai and none of them has marathi mother
tongue. Q works in Hyderabad and his mother tongue is telungu. The
one who works in Delhi has bangle mother tongue. V works in
Mumbai and his mother tongue is hindi, T does not work in Kolkata.

1. What is R’s mother tongue ?


A)Tamil
B)Kannada
C)Punjabi
D)Hindi
E)None of these

Answer

Answer – B)Kannada
Explanation :

2. Who works in Delhi ?


A)T
B)U
C)V
D)S
E)None of these

Answer

Answer – A)T
3. What is U’s mother tongue ?
A)Telungu
B)Hindi
C)Tamil
D)Marathi
E)None of these

Answer

Answer – C)Tamil

4. Who works in Chennai ?


A)S
B)R
C)Q
D)P
E)None of these

Answer

Answer – D)P

5. Which of the following is correct ?


A) Q – Hyderabad – Punjabi
B) T – Ahmedabad – Bangla
C) V – Mumbai – Hindi
D) S – Delhi – Telungu
E)None of these

Answer

Answer – C) V – Mumbai – Hindi

II.Study the following information carefully to answer the questions


given below.
A, B, C, D, E, F and G are 7 friends. Each of them likes a
particular fruit and city. The choices of fruits and favourite city not
necessarily in the same order.
Fruit: Apple, Banana, Pear, Guava, Orange, Mango, Waterlemon
City : Mumbai, Pune, Delhi, Kolkata, Chennai, Hyderabad, Cochin
B likes Mango and her favourite city is Chennai, The one whose
favourite city is pune, likes waterlemon. E’s favourite city is Kolkata.
C likes guava and his favourite city is not Mumbai. G’s favourite city
is cochin and he does not like either banana or pear. The favourite
city of the who likes orange is Hyderabad. E does not like pear. A’s
favourite city is neither pune nor Hyderabad. D does not like
waterlemon.

1. What is F’s favourite city ?


A)Delhi
B)Mumbai
C)Pune
D)Kolkata
E)None of these

Answer

Answer – C)Pune

2. Who like Pear ?


A)C
B)F
C)B
D)A
E)None of these

Answer

Answer – D)A

3. Which fruit does G likes ?


A)Apple
B)Banana
C)Waterlemon
D)Orange
E)None of these
Answer – A)Apple

4. Which of the following combination is correct ?


A) D Kolkata Apple
B) E Hyderabad Banana
C) G Cochin Orange
D) A Mumbai Pear
E)None of these

Answer

D) A Mumbai Pear

5. Which of the following combination is not correct ?


A)B Chennai Mango
B)F Pune Watermelon
C)E Kolkata Banana
D)C Cochin Apple
E)None of these

Answer-D)C Cochin Apple

Study the following information carefully to answer the questions


given below.
Seven specialist doctors B,M,K,P,D,F and H visit a polyclinic on four
days- Tuesday, Wednesday, Friday and Saturday – In a week. At
Least one doctor but not more than two doctors visits the Polyclinic
on each of these days. Each of One is Specialist in different Fields –
ENT, Orthopaedics, Paediatrics, Neurology, Ophthalmology,
Radiology and Oncology.

 P visit on Friday with Radiologist.


 The Paediatrician does not visit on Saturday nor with D and H.
 The Oncologist F visit alone on Tuesday.
 M visits on Wednesday and he is not Paediatrician.
 K visit on Wednesday. H is not Radiologist.
 The Paediatrician visits with the ENT specialist.
 The Neurologist visits on Friday.
 B is neither Orthopaedician nor Radiologist.

1. What is the Speciality of B?


1)ENT
2)Ophthalmology
3)Paediatrics
4)Data inadequate
5)None of these

Answer-2)Ophthalmology

2. On which day of the week does D visit?


1)Wednesday
2)Saturday
3)Friday
4)Wednesday or Saturday
5)None of these

Answer-3)Friday

3. Who among them visits the Polyclinic along with B?


1) H
2)D
3)P
4)Either H or P
5)None of these

ANS- 1) H

4. What is P’s Profession?


1)ENT
2)Paediatrician
3)Ophthalmologist
4)Data Inadequate
5)None of these

Answer- 5)None of these

5. On which of the following days do the specialists in


Orthopaedics and Ophthalmology visit?
1)Wednesday
2)Friday
3)Saturday
4)Data inadequate
5)None of these

Answe-3)Saturday

II.Study the following information carefully to answer the questions


given below.
Seven candidates Harish, Samir, nilesh, shailaja, Nikita, Laxman and
Sujata are to be interviewed for selection as Trainee officers by
different panels 1 to 7 for different banks Bank of India, SBI, Bank of
Maharashtra, PNB, Axis, HDFC and ICICI not necessarily in the same
order.

Nilesh is interviewed by panel 4 for Bank of India. Samir is


interviewed by panel 3 but not for PNB or Bank of Maharashtra.
Harish is interviewed for SBI but not by panel 1 or 2. Nikita is
interviewed by panel 6 for Axis Bank. Panel 7 conducts the interview
for HDFC. Shailaja is interviewed by panel 1 but not for Bank of
Maharashtra. Panel 2 does not interview Laxman.

1. Shailaja is interviewed for which Bank?


1)Bank of India
2)ICICI
3)HDFC
4)PNB
5)None of these

Answer

Answer – 4)PN

2. Panel 2 conducts interview for which Bank?


1)Bank of Maharashtra
2)HDFC
3)ICICI
4)SBI
5)None of these

Answer

Answer – 1)Bank of Maharashtra

3. Who is interviewed for ICICI?


1)Nikita
2)Shailaja
3)Laxman
4)Samir
5)None of these

Answer

Answer – 4)Samir

4. Which candidate is interviewed by panel 5?


1)Laxman
2)Sujata
3)Shailaja
4)Harish
5)None of these

Answer

Answer – 4)Harish

Study the following information carefully to answer the questions


given below.

Eight persons A,B,C,D,E,F,H and I are going to three different


destinations Gujarat, Amritsar, Hyderabad in three different cars-
Chevrolet, Ford, Hyundai. Out of these three are females and also
one in each car. At Least two persons are there in each car.

 E, a male, is travelling with only I and they are not going to


Amritsar.
 A is travelling in Chevrolet and and is going to Hyderabad. C
is not travelling with B and H.
 C and F are travelling together. H is not going to Amritsar. D
is the sister of A and is travelling by Hyundai.

1. Members of which cars are going to Amritsar?


1)Ford
2)Can’t be determined
3)Hyundai
4)Chevrolet

Answer

Answer – 3)Hyundai

2. In which car are four members travelling?


1)None
2)Hyundai
3)Ford
4)Chevrolet

Answer

Answer – 1)None

3. Which of the following combinations represents the three


female members?
1)ABC
2)CID
3)CFE
4)Can’t be determined

Answer

Answer – 4)Can’t be determined

4. Who is travelling with H?


1)CF
2)SI
3)AB
4)Can’t be determined

Answer

Answer – 3)AB

5. Members of which of the following are travelling in


Chevrolet?
1)ACH
2)ABI
3)HAB
4)None of these

Answer

Answer – 3)HAB

II.Study the following information carefully to answer the questions


given below.

 Five friends A,B,C,D and E wore shirts of Green,


Yellow, Pink, Red and Blue colors and shorts of
Black, White, Grey, Blue and Green colors.
 Nobody wore shirt and short of same color.
 D wore Blue shirt and C wore Green Short.
 The one who wore Green shirt, wore Black short
and the one who wore Blue short wore Red shirt
 A wore White short and Pink shirt.
 E did not wear Red shirt.

1. Which color shirt did C wear?


1)White
2)Green
3)Yellow
4)None

Answer

Answer – 3)Yellow
2. Who wore Black Short?
1)B
2)E
3)A
4)C

Answer

Answer – 2)E

3. Who wore White short?


1)C
2)E
3)D
4)A

Answer

Answer – 4)A

4. Which color short did B wear?


1)Grey
2)Black
3)Green
4)Blue

Answer

Answer – 4)Blue

5. If C wore Green shirt and E wore Yellow shirt and if C wore


Pink shirt then what color shirt did A wear?
1)Green
2)Blue
3)Yellow
4)None of these

Answer
Answer – 3)Yellow

Study the following information carefully to answer the question


given below

P, Q, R, S, T, U and V are 7 football players each playing for a different team


– Green , Red and Blue with at least two of them in each of these teams.
Each of them likes a fruit – Apple, Guava, Banana, Orange, Mango, Papaya
and Waterlemon not necessarily in the same order.

Q plays with T in team blue and he likes mango. None of those who play for
either team red or green likes either guava or banana. S plays with only the
one who likes Watermelon. U likes papaya and he plays in team red. The one
who likes orange does not play in team red. V likes watermelon and he plays
for team green. P likes Apple and he plays for team red. R does not like
guava.

1. Which of the following players play for team Red ?


1.SR
2.PU
3.ST
4.PQ
5.None of these

Answer & Solution

Answer – 2.PU

2. Who likes Guava ?


1.T
2.Q
3.S
4.U
5.None of these

Answer

Answer – 1. T

3. Which fruit does S likes ?


1.Mango
2.Banana
3.Apple
4.Orange
5.None of these

Answer

Answer – 4.Orange

4. In which team do 3 of them play ?


1.Blue or Red
2.Red
3.Blue
4.Data inadequate
5.None of these

Answer

Answer – 3.Blue
5. Which of the following combination is incorrect ?
1.T – Blue – Guava
2.U – Red – Papaya
3.Q – Blue – Mango
4.S – Green – Watermelon
5.None of these

Answer

Answer – 4. S – Green – Watermelon

II. Study the following information carefully to answer the question


given below

Seven persons M, N, O, P, Q, R and S are standing in a straight line facing


north at equal distance but not necessarily in the same order. Each of them
is a different professional – Actor, Reporter, Doctor, Engineer, Lawyer,
Teacher and Painter but not necessarily in the same order.

S is standing at the fifth position to the left of O. Reporter is standing at the


third position to the right of S. R is standing at the fifth position to the right
of M. Q is standing second to the left of N. Engineer is standing the second
position to the left of P. Three persons are sitting between Engineer and
Painter. Doctor is to the immediate left of Engineer. Lawyer is to the
immediate right of teacher.

1. Who among the following is sitting second to the right of


Teacher ?
1.O
2.N
3.Lawyer
4.Actor
5.None of these

Answer & Solution

Answer –2.N

2. Who among the following are the immediate neighbours of


Painter ?
1.Doctor and Lawyer
2.Actor and Lawyer
3.Reporter and Actor
4.Lawyer and Reporter
5.None of these

Answer

Answer – 3. Reporter and Actor

3. Who among the following is sitting exactly in the middle of


the row ?
1.R
2.Lawyer
3.O
4.Reporter
5.Teacher

Answer & Solution

Answer – 2.Lawyer

4. Who is sitting at the left end of the row ?


1.M
2.N
3.O
4.P
5.None of these
Answer

Answer – 1.M

5. How many persons are there to the left of Reporter ?


1.One
2.Two
3.Three
4.Four
5.None of these

Answer

Answer – 4.Four

.Study the following information carefully to answer the given


questions
There are 7 friends M, N, O, P,Q, R and S. All of them have a different types
of Car – Alto, Swift, BMW, Audi, Fiat, Nano and WaganR, but not necessarily
in the same order. All of them are using different Service provider – Airtel,
Aircel, Reliance, Vodafone, Idea, Tata Docomo and MTNL.

P has BMW and he used Airtel. The one who has Nano uses Tata Docomo. M
does not use Vodafone, while N who has Swift and uses Idea. Q and R do not
have Nano. Q uses Reliance and WagonR, while the person who has Audi
uses vodafone. O uses MTNL. S who does not own Alto uses Aircel.

1. P uses which of the following Service ?


A.Airtel
B.Aircel
C.Idea
D.Reliance
E.None of these

Answer & Explanation

Answer – A.Airtel

Who had following has Audi ?


A.P
B.R
C.S
D.Q
E.None of these

Answer

Answer – B.R

2. Who among the following uses the Service Aircel ?


A.O
B.N
C.S
D.M
E.None of these

Answer

Answer – C.S

3. Which of the following combination is incorrect ?


A.S-Fiat-Aircel
B.P-BMW-Airtel
C.M-Nano-Tata Docomo
D.R-Alto-Idea
E.None of these

Answer
Answer – D.R-Alto-Idea

4. Which of the following combination is correct ?


A.M-BMW-Reliance
B.S-Swift-Idea
C.N-Swift-Idea
D.R-Alto-Aircel
E.None of these

Answer

Answer – C.N-Shift-Idea

II.Study the following information carefully to answer the given


questions
Arun, Beema, Carolin, Diya, Easwar, Francis and Govind are seven students
of a school. Each of them studies in a different standard I to VI but not
necessarily in the same order. Each of them likes a different Subject –
English, Hindi, Maths, Biology, Chemistry, GK and Computer but not
necessarily in the same order.

Beema studies in Standard VI but he does not like Biology or computer.


Carolin like English and does not study in standard IV or II. Easwar studies
in Standard VII and likes Hindi. The one who likes GK studies in standard I.
Diya studies in standard III. Govind likes Maths. Arun does not study in
standard I.The one who like computer studies in standard IV.
1. Govind studies in which of the following standard ?
A.VI
B.II
C.III
D.V
E.None of these

Answer & Explanation

Answer – B.II

2. Which of the following subjects does Arun likes ?


A.English
B.Biology
C.Computer
D.Hindi
E.None of these

Answer

Answer – C.Computer

3. Who among the following likes Biology ?


A.Diya
B.Beema
C.Carolin
D.Arun
E.None of these

Answer

Answer – A.Diya

4. Which of the following combination is incorrect ?


A.Govind II Maths
B.Diya III Biology
C.Arun IV Computer
D.Francis V English
E.None of these
Answer

Answer – D. Francis V English

5. Carolin studying in which standard ?


A.III
B.IV
C.V
D.VII
E.None of these

Answer

Answer – C.V

I.Study the following information carefully to answer the given


Questions

A, B, C, D, E, F, G and H live on 8 different floors of a buildings but not


necessarily in the same order. The lower most floor of the building numbered
1, the one above that is numbered 2 and so on.Each of them lives on a
different floor also likes a different sports – Badminton, Volley ball, Race,
Cricket, Tennis, Hockey, Football and Boxing.
 The one who likes Tennis lives on an even numbered floor. Only 3
people live between the one who likes Tennis and C.Only 2 people
live between C and D. D does not live on the lowermost floor.Only 3
people live between D and the one who likes Hockey.
 E lives immediately above A. E lives on an even numbered floor. A
does not like Hockey.A lives neither on the floor 3 nor 5.Only 2
people live between A and the one who likes Football.Only one
person lives between the one who likes Football and Volleyball.The
onewho likes Volleyball lives below the one who likes Foot ball.
 B lives immediately above G. Only one persons lives between G and
the one who likes Cricket.The one who likes Race lives immediately
above the one who likes Badminton.F does not like Tennis. A does
not like Volleyball.

1. Which of the following game does H likes ?


A.Hockey
B.Tennis
C.Cricket
D.Race
E.None of these

Answer & Explanation

Answer – B.Tennis

2. D lives on the which of the following floor numbers ?


A.5
B.4
C.3
D.6
E.None of these

Answer

Answer – A.5

3. Who among the following likes Hockey ?


A.H
B.E
C.C
D.F
E.None of these

Answer

Answer – D.F

4. Which of the following pair is correct ?


A.E-Race
B.D-Boxing
C.G- Tennis
D.A – Football
E.one of these

Answer

Answer – A.E-Race

5. Who lives below the person one who likes Cricket ?


A.F
B.H
C.B
D.C
E.None of these

Answer

Answer – C.B
II.Study the following information carefully to answer the given
Questions

K, L, M, N, O, P and Q are seven members of a family. Each of them has a


different profession-Doctor, Teacher, Lawyer, Engineer, Architect, CA and
banker, their incomes are different.

 There are 2 married couples in the group.M is the doctor and earns
more than Engineer and the Lawyer. O married to the CA and she
earns the least. No lady is either lawyer or engineer.
 L, the teacher earns less than K, the lawyer Q is married L, he earns
more than N and K. P is not the lawyer. The CA earns less than
lawyer but more than the banker.

1. What is the profession of P ?


A.Lawyer
B.CA
C.Doctor
D.Engineer
E.None of these

Answer & Explanation

Answer –B.CA

2. Who earns the minimum in the family ?


A.Teacher
B.Lawyer
C.Banker
D.Architect
E.None of these

Answer

Answer – D.Architect

3. Which of the following pair is a married couple ?


A.ON
B.KM
C.LQ
D.QN
E.None of these

Answer

Answer – C.LQ

4. Who among the following is husband of O ?


A.L
B.P
C.M
D.N
E.None of these

Answer

Answer – B.P

5. Which of the following is not correct ?


A.Q – Lawyer – M
B.P – CA-M
C.L – Teacher – F
D.N – Engineer – M
E.None of these

Answer

Answer – B.P – CA-M


.Study the following information carefully to answer the given
Questions

Eight groups P, Q, R, S, T, U, V and W are going for rock climbing in different


mountain ranges, viz. Saser Kangri, Rimo, Aravali, Chaukhamba, Nanda
Devi, Hardeol, Trisul and Kamet but not necessarily in the same order. These
groups are also going for rafting in different rivers, viz. Meghna, Yamuna,
Tons,Mahanadi, Tabi, Narmada, Koshi and Ganges but not necessarily in the
same order,

 Group V is going neither to river Koshi nor to river Yamuna, Neither


group U nor group V is going to raft in the river Mahanadi or the
river Narmada. Group T is going to the Kamet mountain range.
 Group R is going to the Chaukhamba mountain range and the river
Tabi. The one who is going to raft in the Tons is also going to the
Kamet mountain range. Group P and Q are going for rafting either in
the Narmada or Ganges.
 Group S does not want to raft in the Mahanadi and U does not want
to raft in the Yamuna. The group which is climbing on the Sasar
Kangri is rafting in the Koshi and the group which is climbing on the
Trisul is rafting in the Yamuna. P and V do not climb on the Hardeol
and the Aravali.The group which is climbing on the Rimo is rafting in
the Ganges. Group W and V climb either on Nanda Devi or on the
Aravali mountain range.

1. Which group is going to raft in the river Narmada?


1.P
2.Q
3.R
4.S
5.None of these

Answer & Explanation

Answer – 2.Q
2. Which of the following groups is going to climb on the Sasar
Kangri ?.
1.S
2.T
3.U
4.W
5.None of these

Answer

Answer – 3.U

3. The group rafting in the Tons is climbing in which of the


following mountain ranges?.
1.Nanda devi
2.Trisul
3.Rimo
4.Kamel
5.None of these

Answer

Answer – 4.Kamel

4. The group Q is going to climb on the which of the following


mountain ?
1.Hardeol
2.Trisul
3.Rimo
4.Aravali
5.None of these

Answer

Answer – 1.Hardeol

5. Which of the following is true ?


1.P – Tabi
2.S – Trisul
3.V – Nanda Devi
4.R – Ganges
5.None of these

Answer & Explanation

Answer – 3.V – Nanda Devi

II.Study the following information carefully to answer the given


questions

Eight persons A, B, C, D, E, F, G and H belonging to different cities viz Delhi,


J & K, MP, Kerala, AP, Odisha, Rajasthan and Tamilnadu, but not necessarily
in the same order, went to Hyderabad to attend an Education programme
.Each had a different specialisation, viz Literature, Science, Presentation,
Advertisement, Designing, Information Technology, Drawing and Writing, but
not necessarily in the same order.

 E is from AP. G does not have specialisation in Literature or Science.


C is from MP and he has specialisation in Designing. Neither F nor G
has specialisation in Advertisement or Information Technology. One
who is from AP has specialisation in Drawing. A and B have
specialisation either in Information Technology or in Writing.
 D does not have specialisation in Advertisement and F does not
have specialisation in Science. One who is from Tamilnadu has
specialisation in Literature and one who is from Kerala has
specialisation in Science. G and H belong either to Rajasthan or to
Odisha.
 A and G do not belong to J & K and Rajasthan respectively. One who
belongs to Delhi has specialisation in Writing.

1. The person with specialisation in Information Technology


comes from which of the following cities?
1.MP
2.J & K
3.Kerala
4.Delhi
5.None of these

Answer & Explanation

Answer – 2.J & K

2. Which of the following persons is from Tamilnadu ?


1.G
2.H
3.F
4.C
5.None of these

Answer

Answer – 3.F

3. Who is specialised in Science ?


1.A
2.B
3.C
4.D
5.None of these

Answer & Explanation

Answer – 4.D

4. Which of the following is true ?


1.H – AP
2.G – Tamilnadu
3.A – Delhi
4.F – Odisha
5.None of these

Answer

Answer – 3.A – Delhi


5. Which of the following is not true ?
1.H – Presesentation
2.C – Designing
3.E – Drawing
4.D – Science
5.None of these

Answer

Answer – 1. H – Presesentation

Study the following information carefully to answer the given


questions

Seven friends A, B, C, D, E, F and G appear for GATE exam on different dates


in December viz 5th, 6th, 12th, 13th, 19th, 25th and 26th but not necessarily in the
same order. Each of them belongs to a different cities viz Delhi, Mumbai,
Chennai, Hyderabad, Ranchi, Amritsar and Kolkata but not necessarily in the
same order.

The one who belongs to Delhi appears in the exam on 25th Feb. G belongs to
Mumbai and appears in the exam on 12th Feb. B appears in the exam
immediately before E. B does not appear in the exam on a day after G. The
one who belongs to Kolkata does not appear in the exam on or before
19th Feb. The one who belongs to Chennai appears in the exam immediately
after C. E does not belong to Ranchi. The one who belongs to Amritsar does
not appear in the exam immediately before or after G. F does not appear in
the exam on 26thFeb nor does he belong to Chennai. D appears in the exam
on a day before A.

1. Who among the following belongs to Ranchi ?


A.A
B.B
C.C
D.D
E.None of these

Answer & Explanation

Answer –C

2. F belongs to which of the following cities ?


A.Delhi
B.Mumbai
C.Chennai
D.Ranchi
E.None of these

Answer & Explanation

Answer – A.Delhi

3. On which day does B appear in the exam ?


A.Feb 5th
B.Feb 19th
C.Feb 13th
D.Feb 26th
E.None of these

Answer

Answer – A.Feb 5th

4. Which of the following is true ?


A.D-Feb 6th
B.E-Mumbai
C.G-Feb 12th
D.A-Chennai
E.None of these

Answer
Answer – C.G-Feb 12th

5. Who wrote the exam on Feb 26th ?


A.D-Delhi
B.E-Kolkata
C.G-Mumbai
D.A-Kolkata
E.None of these

Answer

Answer – D.A-Kolkata

II.Study the following information carefully to answer the given


questions

A family of seven persons M, N, O, P, Q, R and S are go for a picnic in Tata


Sumo. There are 2 couples in the family. Only 2 members can sit in the front
row as well as in the back row of the car and rest can sit in the middle row.
Not more than 2 males or females can sit in the same row or column. The
daughter-in-law is driving the car. One couple is seated in the same column.
The granddaughter sits on the immediate left of her grandmother.

N is father of Q. S has 2 sons and a daughter. R sits in the back row. N and P
doesn’t sit in the front row. O is married to N.M is father of R. Both the
brothers not sit in the same column.P is daughter of M.

1. How many male members are there in the family ?


A.Three
B.Four
C.Two
D.Can’t be determined
E.None of these

Answer & Explanation

Answer – A.Three
2. Who among the following is mother of N ?
A.M
B.O
C.S
D.R
E.None of these

Answer

Answer – C.S

3. Which of the following group sits in the middle row of the car
?
A.R,S
B.M,N,O
C.O,P
D.Q,S,N
E.None of these

Answer

Answer – D.Q,S,N

4. Who among the following is father of Q ?


A.N
B.P
C.Q
D.R
E.None of these

Answer

Answer – A.N

5. Which is true with respect to the given arrangement ?


A.P-middle
B.O-front
C.M-back
D.S-front
E.None of these

Answer

Answer – B.O-front

Study the following information carefully to answer the given


questions

Seven friends P, Q, R, S, T, U and V belongs to 7 different states viz UP,


Delhi, AP, Rajasthan, Punjab, Kerala and Gujarat but not necessarily in the
same order. Each of them likes different dishes viz Dosa, Burger, Rajma,
Pizza, Sandwich, Samosa and Dhokla but not necessarily in the same order.

Q likes neither Dosa nor Dhokla. S belongs to AP. R belongs to Delhi and
likes Pizza. T and U do not hail from Rajasthan. The one who hail from
Rajasthan likes Burger. Neither Sandwich nor Dosa is liked by S. Q hails from
Punjab. The one who likes Dhokla is either from Gujarat or from Punjab. U
does not like Dosa or Sandwich. P likes Rajma. V is not from Kerala.

1. The one who likes Samosa is from which of the following


states ?
1.UP
2.Kerala
3.AP
4.Gujarat
5.None of these

Answer & Explanation 3.AP


2. T likes which of the following dishes ?
1.Burger
2.Pizza
3.Dhokla
4.Dosa
5.None of these

Answer

Answer – 4.Dosa

3. If the person who likes Dosa is from UP then the person who
likes Rajma from which of the following state ?
1.Delhi
2.AP
3.Kerala
4.Rajasthan
5.None of these

Answer

Answer – 3.Kerala

4. Who among the following like Dhokla ?


1.V
2.U
3.T
4.R
5.None of these

Answer

Answer – 2.U

5. Which of the following is true ?


1.U – Gujarat – Dhokla
2.Q – UP – Dosa
3.R – AP – Burger
4.S – Delhi- Rajma
5.None of these

Answer

Answer – 1. U – Gujarat – Dhokla

P – UP/Kerala – Rajma
Q – Punjab – Sandwich
R – Delhi – Pizza
S – AP – Samosa
T – UP/Kerala – Dosa
U – Gujarat – Dhokla
V – Rajasthan – Burger

II.Study the following information carefully to answer the given


questions

Seven friends G, H, I, J, K, L and M meet in a party. They shake hands with


each other but only in a particular way. The odd numbered persons do not
shake hands with even numbered persons and vice versa. These friends are
wearing shirts of different colours such as Red, White, Yellow and Black. No
two even numbered persons can wear the shirts of the same colour and the
same rule not applied for odd numbered persons.All the persons shake hands
with at least one person.

K is the only person who shakes hands with all odd numbered persons but he
is not wearing a Black shirt. L is wearing a Red shirt and shakes hands only
with H. The persons who wear Yellow shirts shake hands with only the
persons who wear White shirts. G does not wear a Black shirt but shakes
hands with the persons who wear a Black shirt. Black shirt is worn by only
one person but that person is not I.

1. Who among the following wear White shirt ?


1.G and I
2.H and K
3.I and M
4.J and L
5.None of these

Answer & Explanation

Answer – 2.H and K


G wears shirt which of the following colour ?
1.Yellow
2.Black
3.White
4.Red
5.Can’t be determined

Answer

Answer – 4.Red

2. Who among the following wear Black shirt ?


1.M
2.I
3.H
4.L
5.None of these

Answer

Answer – 1.M

3. K hand shakes with who among the following ?


1.G and M
2.H and L
3.G, I and M
4.K, L and G
5.None of these

Answer

Answer – 3.G, I and M


4. 5.Which of the following is correct ?
1.L – Black – Shake hands with G, M
2.G – Red shirt – Shake hands with K, M
3.I – Yellow- Shake hands with K
4.M – White – Shake hands with L
5.None of these

Answer

Answer – 2.G – Red shirt – Shake hands with K, M

Explanation :
G – Red shirt – Shake hands with K, M
H – White – Shake hands with J, L
I – Yellow – Shake hands with K
J – Yellow – Shake hands with H
K – White – Shake hands with G,I, M
L – Red – Shake hands with H
M – Black – Shake hands with G, K

Study the following information carefully to answer the given


questions

Eight persons A, B, C, D, E, F, G and H are live in a on separate floors in an 8


storeyed building in Delhi. Each floors are numbered, Ground floor as number
1 and topmost as number 8.

 E lives on an even numbered floor and immediately below the floor


where F lives.
 A lives below the floor H lives in. B lives on floor number 6.
 Exactly one person lives between floor H and floor A.
 C lives on lowermost floor. A lives immediately below floor B lives
in.
 D does not live on topmost floor.

1. On which floor number does D live?


A. 3
B. 2
C. 4
D. None

Answer & Explanation

C. 4

2. Who lives on floor number 6?


A. E
B. C
C. D
D. B

Answer & Explanation

D. B

3. How many persons live in between E and B?


A. 4
B. 2
C. 3
D. 1

Answer & Explanation

C. 3

4. The person who lives immediately below H?


A. B
B. C
C. D
D. A
Answer & Explanation

A. B

Explanation:
Floor 8 G
Floor 7 H
Floor 6 B
Floor 5 A
Floor 4 D
Floor 3 F
Floor 2 E
Floor 1 C

II.Study the following information carefully to answer the given


questions
Eight friends Puneet, Rasika, Amit, Shilpi, Tanvi, Kiran, Nav and Navneet are
sitting around a square table such a that four of them sit at four corners of
the square while four sit in the middle of each of the four sides. Persons who
sit at the four corners face the centre while those who sit in the middle of the
sides face outside.

 Shilpi sits third to the right of Puneet. Puneet faces the centre.
 Navneet is not an immediate neighbor of Puneet or Shilpi. Tanvi sits
third to the right of Amit.
 Amit does not sit in the middle of any of the sides and is not an
immediate neighbor of Navneet.
 Only one person sits between Puneet and Kiran. Rasika is not an
immediate neighbor of Kiran.

1. If all the persons are made to sit in alphabetical order in


clockwise direction, starting from Amit, the positions of how
many (excluding Amit) will remain unchanged as compared
to their original seating positions?
A. None
B. One
C. Two
D. Three

Answer & Explanation

B. One
Explanation:Puneet

2. Which of the following is true regarding Navneet?


A. Tanvi is not immediate neighbor of Navneet
B. Navneet sits in the middle of one of the sides
C. Amit sits second to left of Navneet
D. P and V are immediate neighbors of Navneet

Answer & Explanation

C. Amit sits second to left of Navneet

3. Who amongst the following sits fourth to the left of Kiran?


A. Navneet
B. Amit
C. Tanvi
D. Rasika

Answer & Explanation

A. Navneet

4. What is the position of Rasika with respect to Amit?


A. Immediately to the left
B. Immediately to the right
C. Third to the left
D. Third to the right

Answer & Explanation

B. Immediately to the right

5. Four of the following five are alike in a certain way and so


form a group. Which is the one that does not belong to the
group?
A. Tanvi
B. Nav
C. Kiran
D. Shilpi

Answer & Explanation

C. Kiran

6. Who sits third to the right of Nav?


A. Amit
B. Shilpi
C. Rasika
D. Navneet

Answer & Explanation

D. Navneet

7. How many people sit between Tanvi and Rasika?


A. None
B. One
C. Two
D. Three

Answer & Explanation

B. One

Study the following information to answer the given questions

Seven friends A, B,C,D,E,F and G attend different language classes namely


French, English, Spanish, Sanskrit, Chinese,German and Japanese but not
necessarily in the same order from Monday to Sunday.
 A attends a class on Friday. Only 2 persons attend classes between
A and the one who is learning Sanskrit. B attends a class
immediately after E. Neither F nor D is learning Sanskrit. Only one
person attends a class between F and the one who is learning
Spanish.
 The one who is learning Spanish does not have a class on Monday.
B attends a class immediately before the one who is learning
French. A is not learning French. Only one person has a class
between F and E. C is learning German. Only 2 persons have classes
between C and the one who is learning Chinese. E is not learning
Japanese

1. Which of the following language is E learning ?


1.Japanese
2.English
3.German
4.French
5.None of these

Answer & Explanation

Answer – 2.English

2. If G have class on Saturday .On which of the following day


does D have a class ?
1.Thursday
2.Wednesday
3.Monday
4.Tuesday
5.None of these

Answer

Answer – 1.Thursday

3. Who among the following is learning Japanese ?


1.E
2.F
3.G
4.A
5.None of these

Answer

Answer – 3.G

4. Who among the following have classes immediately before B


?
1.G
2.C
3.A
4.E
5.None of these

Answer

Answer – 4.E

5. Which of the following is correct ?


1.D-Tuesday-Sanskrit
2.A-Friday-Spanish
3.G-Sunday-Japanese
4.B-Monday-English
5.None of these

Answer

Answer – 2.A-Friday-Spanish

Explanation :
A-Friday-Spanish
B-Tuesday-Sanskrit
C-Sunday-German
D/G-Thursday-Chinese
E-Monday-English
F-Wednesday-French
G/D-Saturday-Japanese
II.Study the following information to answer the given questions

P,Q,S,T,U,V and W live on a different floor of a building having seven floors


numbered 1 to 7, Ground floor is numbered 1 and so on. All of them like a
different colours viz Yellow, Black, Green, Orange, Red, White and Pink but
not necessarily in the same order.

 There are 2 floors between the floor on which Q and U live. Q lives
two floors below W’s floor and likes Green Colour. There are 3 floors
between W and P. P likes Yellow colour and lives on an odd
numbered floor. The one likes Pink colour lives on floor no 6. The
one who likes Yellow lives on floor no 3.
 The one who likes Orange colour does not live on an even numbered
floor. Neither S nor U lives immediately above or immediately above
or immediately below T’s floor. S likes white colour and there is one
floor between the floor on which T and S live. T lives on an even
numbered floor but he does not live on floor no 4. P lives
immediately above U’s floor. V likes Black colour but he does not
live on the topmost floor. U likes Red colour and lives immediately
below P’s floor.
 There is only one person between the person who likes Pink colour
and the one who likes white colour but that person is neither P nor
U. The one who likes Orange colour lives immediately above T’s
floor. T does not live on floor no 2. Those who live on floor no 4 and
floor no 2 like White and Red colours respectively.

1. On which of the following floor does V live ?


1.First
2.Fourth
3.Sixth
4.Seventh
5.None of these

Answer & Explanation

Answer – 1.First
2. U likes which of the following colour ?
1.Pink
2.Red
3.Green
4.white
5.None of these

Answer

Answer – 2.Red

3. Who among the following lives on the fourth floor ?


1.S
2.P
3.W
4.U
5.None of these

Answer

Answer – 1.S

4. How many floors are there between the floors P and T ?


1.Four
2.Five
3.One
4.Two
5.None of these

Answer

Answer – 4.Two

5. Which of the following is true ?


1.5-V-White
2.1-U-Black
3.7-W-Orange
4.6-S-Green
5.None of these
Answer

Answer – 3.7-W-Orange

Explanation :
1-V-Black
2-U-Red
3-P-Yellow
4-S-White
5-Q-Green
6-T-Pink
7-W-Orange

Study the following information to answer the given questions

There are 7 friends P, Q, R, S, A, B and C live in three different colonies – X,


Y and Z. A minimum of two or a maximum of three friends live in one colony.
Each of them is wearing shirts of different colours i.e. red, green, blue,
white, black, yellow and pink but not necessarily in the same order. B is
wearing a shirt of pink colour and lives in the same colony as only C i.e. X. R
is wearing a black shirt and does not live in building Z. S doesn’t live in the
same building as P or A and is wearing a yellow colour shirt. Q lives in
building Z with only one more person and is wearing a green shirt. None in
the Y building is wearing a white shirt. A doesn’t wear a blue shirt.

1. Who live in colony Z ?


1.P,Q
2.Q,R
3.Q,S
4.B,C
5.None of these

Answer
Answer – 3.Q,S

2. Who is wearing black white shirt?


1.B
2.C
3.S
4.Can’t be determined
5.None of these

Answer

Answer – 2.C

3. Who is living in colony Y?


1.PRQ
2.PRA
3.ABC
4.ARQ
5.None of these

Answer

Answer – 2.PRA

4. which of the following is true?


1.P – Z – PINK
2.R – Y – WHITE
3.C – X – WHITE
4.S – Z – BLACK
5.None of these

Answer & Explanation

Answer – 3.C – X – WHITE

5. A live in which colony and wears which colour shirt-


1.Z – RED
2.Y – WHITE
3.X – RED
4.Y – RED
5.None of these

Answer

Answer – 4.Y – RED

P Y BLUE
Q Z GREEN
R Y BLACK
S Z YELLOW
A Y RED
B X PINK
C X WHITE

II.Study the following information to answer the given questions

P, Q, R, S, T, U and V are seven friends travelling in 3 different vehicles i.e.


ford, maruti and Hyundai. At least two friends travel in one car. Among them
two are engineers, two are doctors and three are teachers. Every car has
persons of both sexes. R is a lady doctor and she does not travel with the
pair of sisters P and U. Q a male engineer, travels with only V, a teacher in
ford. S is a male doctor. Two persons belonging to the same profession does
not travel in the same vehicle. P is not an engineer and travels in maruti

1. How many females are there in the group?


1.2
2.3
3.4
4.5
5.None of these

Answer

Answer – 3.4
2. Which of the following represents the three teachers?
1.PQR
2.PTU
3.PTV
4.STV
5.None of these

Answer

Answer – 3.PTV

3. What is U’s profession?


1.doctor
2.teacher
3.engineer
4.can’t be determined
5.None of these

Answer

Answer – 3.engineer

4. Who travels in maruti vehicle?


1.PRS
2.PSU
3.STU
4.STV
5.None of these

Answer

Answer – 2.PSU

5. Find out the correct one


1.P – MALE – ARUTI – DOCTOR
2.R – FEMALE – FORD – DOCTOR
3.U – FEMALE – MARUTI – TEACHER
4.V – MALE –FORD – TEACHER
5.None of these
Answer

Answer – 5.None of these

P FEMALE MARUTI TEACHER


Q MALE FORD ENGINEER
R FEMALE HYUNDAI DOCTOR
S MALE MARUTI DOCTOR
T MALE HYUNDAI TEACHER
U FEMALE MARUTI ENGINEER
V FEMALE FORD TEACHER

Study the following information to answer the given questions

There are five friend’s akash, bharat, chanda, dinesh and esha. Two of them
are businessman and the other three are painter, artist and teacher. Two of
them live in locality M and other three lives in N, O and P locality. Two of
them are Hindus while other three are Muslim, Sikh and Christian. One
businessman and the teacher stay in the same locality M. The teacher is
oldest in the age while one of the businessmen who run a dairy is the
youngest. The other businessman is diamond merchant and age-wise lies
between the painter and teacher. Dinesh is a diamond merchant and stays in
locality M while Esha is a Muslim and stays in locality P. The painter is
Christian and stays in locality N, Bharat is a Sikh while Akash is a Hindu and
runs a dairy.

Who stays in locality N?


a) Akash
b) bharat
c) chanda
d) dinesh
e) can’t be determined

Answer

Answer – c) chanda
1. What is esha’s profession?
a) Diamond merchant
b) painter
c) teacher
d) artist
e) dairy operator

Answer

Answer – d) artist

2. Age-wise who among the following lies between


chanda and akash?
a) Diamond merchant
b) artist
c) bharat
d) painter
e) dinesh

Answer

Answer – b) artist

3. What is Bharat occupation?


a) Dairy operator
b) artist
c) teacher
d) painter
e) can’t be determined

Answer

Answer – c) teacher

4. Who among the friends belong to hindu religion?


a) Akash, esha
b) akash, bharat
c) bharat, dinesh
d) chanda, dinesh
e) none of these

Answer

Answer – e) none of these

5. If the age of dinesh is 35 years and age of akash is 25


then what is the most probable age of esha?
a) 36
b) 34
c) 29
d) 24
e) None of these

Answer

Answer – c) 29

AKASH BUSSINESS (DAIRY) O HINDU

BHARAT TEACHER M SIKH

CHANDA PAINTER N CHRISTIAN

BUSSINESS
DINESH M HINDU
(DIAMOND)

ESHA ARTIST P MUSLIM

II.Study the following information to answer the given questions


Seven friends P, Q, R, S, T, U and V are standing in a row facing north. T and
Q have two children between them. V is to the right of S and to the left of Q.
S and U have two children between them. P is on the right of R. P and S has
one child between them.
7. Who is on the extreme left?
a) T
b) P
c) U
d) R
e) None of these

Answer

Answer –d) R

8. Who is standing exactly in the middle?


a) P
b) T
c) S
d) Q
e) None of these

Answer

Answer – c) S

9. If the position of P and V are interchanged then who is


second to the left of P?
a) R
b) T
c) U
d) S
e) None of these

Answer

Answer – b) T

10. How many people are standing between S and Q?


a) One
b) two
c) three
d) four
e) none

Answer

Answer – a) One

Study the following information to answer the given questions

There is a family of six members P, Q, R, S, T and U. There are two married


couples in the family and the family is of three generations. Each family
member has different cars Maruti, Honda, Audi, BMW, Mercedes and Bentley.
No lady members have Maruti or Mercedes. R who has Audi is daughter-in-
law of T. Q is the brother of U and son of S and has Bentley. P is the
grandmother of U and U does not have BMW. A husband and a wife have
Maruti and Honda respectively.

1. Which car is maintained by Q?


a) Honda
b) audi
c) Bentley
d) maruti
e) None of these

Answer

Answer – c) Bentley

2. How many male members are there in the family?


a) 2
b) 3
c) 4
d) can’t be determined
e) None of these
Answer

Answer – c) 4

3. Which of the following is true about U?


a) Brother of Q
b) Sister of Q
c) Either brother or sister of W
d) Son of Q
e) None of these

Answer

Answer – a) Brother of Q

4. Which of the following is one of the couples?


a) TS
b) UR
c) PR
d) SR
e) None of these

Answer

Answer – d) SR

5. Which of the following car are maintained by one of the


couple ?
a) Audi-BMW
b) Mercedes – maruti
c) Honda- maruti
d) Honda- Audi
e) Both a and c

Answer

Answer – e) Both a and c

II.Study the following information to answer the given questions


There are seven students in a class P, Q, R, S, T, U and V in a class that
studies different subjects – Hindi, Science, Maths, English, Computer,
Sanskrit and Biology. Each student secured different marks in the exam. R
has secured second highest marks and he doesn’t study hindi or science. T
secured the least marks and he doesn’t like science and Sanskrit. The
favourite subject of S is maths and he secured more marks than Q and V but
less than P. The favourite subject of Q is computer and his marks are more
than T but less than marks of V. The favourite subject of P is biology. The
student whose favourite subject is English got the highest mark

1. Whose favourite subject is English?


a) P
b) R
c) T
d) U
e) None of these

Answer

Answer – d) U

2. Which is the favourite subject of T?


a) English
b) Maths
c) Hindi
d) Biology
e) None of these

Answer

Answer – c) Hindi

3. When all the students are arranged in descending order of


marks, then what is the position of S from the right end?
a) first
b) second
c) third
d) fourth
e) None of these

Answer

Answer – d) fourth

4. How many students are between R and Q in terms of marks?


a) one
b) two
c) three
d) four
e) None of these

Answer

Answer – c) three

5. Who got the second least marks ?


a) P
b) R
c) U
d) Q
e) None of these

Answer

Answer – d) Q

Direction(1-6): Study the following information carefully to answer


the given questions
P, Q, R, S, T, U, V and W are eight members of a family. They belong to
three different generations. There are three married couples. They are all
sitting around a circular table facing the centre but not necessarily in the
same manner. R and P are children of S. P are not an immediate neighbour
of his mother. S and U are married couples. S the wife of U sits second to the
left of her husband. There is only one person sitting between R and his niece
V, but that person is not V’s father. T, a bachelor sits third to the right of his
uncle P but neither to the opposite nor to the immediate left of his father. V
is not an immediate neighbour of her aunt Q. No three females are sitting
together. R and his sister-in-law is immediate neighbour

1. How many females members in the family?


a) three
b) four
c) five
d) can’t be determined
e)None of these

Answer

Answer – b) four

2. What is the relation between U and V?


a) U is the father of V
b) U is the uncle of V
c) U is the grandfather of V
d) U is the nephew of v
e)None of these

Answer

Answer – c) U is the grandfather of V

3. How many persons are seating between P and R when


counted clockwise.
a) one
b) two
c) three
d) four
e)None of these

Answer

Answer – b) two

4. If all the members of the family are arranged in alphabetical


manner starting from P in clockwise direction, then how
many will retain their original position
a) none
b) one
c) two
d) three
e)None of these

Answer

Answer – a) none

5. Four of the following are alike in some manner, find the odd
one.
a) S
b) Q
c) P
d) W
e) V

Answer

Answer – c) P

6. Who among the following is T’s father?


a) W
b) U
c) P
d) R
e)None of these
Answer

Answer – d) R

Direction(7-10) : Study the following information carefully to answer


the given questions

A, B, C, D, E, F, G and H are eight friends sitting around a circular table


facing towards the centre. H is on the immediate left of A but is not the
neighbour of D or E. C is between E and F. F is on the immediate right of B
and G is the neighbour of E

7. What is the position of H?


a) Second to the left of A
b) Fourth to the right of E
c) Third to the left of B
d) Immediate left of D
e)None of these

Answer

Answer – c) Third to the left of B

8. How many people are sitting between A and C.


a) one
b) two
c) three
d) four
e) None of these

Answer

Answer – c) three

9. If all the people are arranged in alphabetical manner starting


from A in clockwise direction, then how many will retain their
original position (excluding A)
a) one
b) two
c) three
d) four
e)None of these

Answer

Answer – a) one

10. What is true about C.


a) H is between E and G
b) H is third to the left of C
c) C and D are sitting opposite to each other
d) none is true
e)None of these

Answer

Answer – d) none is true

irection(1-5): Study the following information carefully to answer the


given questions

Eight persons from different cities – Raipur, Mathura, Allahabad, Jabalpur,


Nagpur, Chandigarh, dehradun and Kanpur, – are sitting in two parallel rows
containing four people each in such a way that there is an equal distance
between adjacent persons. In row 1 A, B, C and D are seated and all of them
are facing north. In row 2, P, Q, R and S are seated and all of them are
facing south. Therefore, in the given seating arrangement, each member is
seated in a row facing another member of the other row.
 C sits second to right of the person from Kanpur. R is an immediate
neighbour of the person who faces the person from Kanpur.
 Only one person sits between R and the person from Jabalpur. The
immediate neighbour of the person from Jabalpur faces the person
from Allahabad.
 The person from Raipur faces the person from Chandigarh. R is not
from Chandigarh. P is not from Jabalpur. P does not face the person
from Kanpur.
 Q faces the person from Nagpur. The one who faces S sits on the
immediate left of A.
 B does not sit at any of the extreme ends of the line. The person
from Kanpur does not face the person from Mathura.

1. Who is sitting between R and the person from Jabalpur ?


a) P
b) S
c) A
d) Q
e) None of these

Answer

Answer – b) S

2. Who among the following sit at the extreme end of line?


a) D and the person from Jabalpur
b) The person from Raipur and dehradun
c) The person from Nagpur and P
d) The person from Mathura and D
e) None of these

Answer

Answer – d) The person from Mathura and D

3. Who among the following facing the person from Kanpur?


a) P
b) Q
c) R
d) S
e) None of these

Answer

Answer – d) S

4. Who among the following is from Chandigarh?


a) A
b) B
c) C
d) D
e) None of these

Answer

Answer – d) D

5. A is from which of the following city?


a) Allahabad
b) Jabalpur
c) Nagpur
d) Kanpur
e) None of these

Answer

Answer – c) Nagpur

Direction(6-10): Study the following information carefully to answer


the given questions

Priya, Neha, Niharika, Pragya, Reshu and Rani live on different floors in the
same building. All of them own one car. Each and each car is of a different
company namely Honda, BMW, Volvo, Land rover, Audi and Mustang but not
necessarily in the same order. The ground floor is numbered 1, the floor
above it number 2, and so on, and the topmost floor is numbered 6. Neha
lives on an even-numbered floor and does not own BMW or Volvo car. Only
two people live between the floors on which Neha and Rani live. Pragya lives
on the floor immediately above the floor on which Niharika lives. Pragya does
not live on an odd-numbered floor and lives immediately below that of one
who owns a Honda car. Priya does not live on the floor immediately above or
immediately below the floor on which Reshu lives. Priya does not live on the
lower-most floor, i.e. floor number 1. Pragya owns Audi car. Priya owns a
Mustang car.

6. Who amongst the following live on the floors between the


floors on which Neha and Rani live?
a) Reshu and Priya
b) Pragya and Reshu
c) Pragya and Niharika
d) Niharika and Priya
e) None of these

Answer

Answer – c) Pragya and Niharika

7. On which of the following floors does Reshu live?


a) 2nd
b) 3rd
c) 4th
d) 5th
e) None of these

Answer

Answer – e) None of these

8. How many people live on the floors above the floor on which
Priya lives?
a) none
b) one
c) two
d) three
e) More than three

Answer

Answer – a) none

9. Who amongst the following lives on floor number 5?


a) Reshu
b) Neha
c) Niharika
d) Priya
e) None of these

Answer

Answer – e) None of these

10. Which car is owned by the person who lives on the 3rd
floor?
a) Volvo
b) Mustang
c) Audi
d) Can’t be determined
e) None of these

Answer

Answer – d) Can’t be determined

Direction(1-5): Study the following information carefully to answer


the given questions

Eight groups P, Q, R, S, T, U, V and W are going for rock climbing in different


mountain ranges, viz. Aravallis, Himalayas, Satpura, Andes, Alps, Rockies,
Atlas and Appalachian but not necessarily in the same order. These groups
are also going for rafting in different rivers, viz. Ganga, Yamuna, Narmada,
Gomti, Krishna, Godavari, Jhelum and Ravi but not necessarily in the same
order.

Group V is going neither to river Jhelum nor to river Yamuna. Neither group
U nor group V is going to raft in the river Gomti or the river Godavari. Group
T is going to the Appalachian mountain range. Group R is going to the Andes
mountain range and the river Krishna. The one who is going to raft’ in the
Narmada is also going to the Appalachian mountain range. Group P and Q
are going for rafting either in the Godavari or Ravi. Group S does not want to
raft in the Gomti and U does not want to raft in the Yamuna. The group
which is climbing on the Aravallis is rafting in the Jhelum and the group
which is climbing on the Atlas is rafting in the Yamuna. P and V do not climb
on the Rockies and the Satpura. The group which is climbing on the
Himalayas is rafting in the Ravi. Group W and V climb either on Alps or on
the Satpura mountain range

1. Which of the following groups is going to climb on the


Rockies?
1) P
2) R
3) Q
4) U
5) None of these

Answer

Answer – 3) Q

2. The group rafting in the Ganga is climbing in which of the


following mountain ranges?
1) Himalayas
2) Rockies
3) atlas
4) alps
5) None of these

Answer

Answer – 4) alps

3. Which group is going to raft in the river Krishna?


1) P
2) U
3) T
4) S
5) None of these

Answer

Answer – 5) None of these

4. Which of the following combinations is not true with respect


to the above information?
1) V- Alps-Ganga
2) U-Aravallis-Jhelum
3) T-Appalachian-Narmada
4) Q-Rockies-Godavari
5) All are true

Answer

Answer – 5) All are true

5. Group P and T are rafting in which of the following rivers


respectively?
1) Ganga and Yamuna
2) Ravi and ganga
3) Narmada and Ravi
4) Ravi and Narmad
5) None of these

Answer
Answer – d) Ravi and Narmad

P HIMALAYAS RAVI
Q ROCKIES GODAVARI
R ANDES KRISHNA
S ATLAS YAMUNA
T APPALACHIAN NARMADA
U ARAVALIS JHELUM
V ALPS GANGA
W SATPURA GOMTI

Direction(6-10): Study the following information carefully to answer


the given questions

Twelve wards P, Q, R, S, T, U, A, B, C, D, E and F of a hospital are in two


parallel buildings. Building 1 is facing south and Building 2 is facing north.
Wards P, Q, R, S, T and U are in Building 1and wards A, B, C, D, E and F are
in Building 2. Each ward of Building 1 is exactly opposite the other ward in
Building 2.

-> Ward C, which is at one of the ends of the building, is second to the left of
ward F
-> There are two wards between ward A and B
-> There is only one ward between ward S and T
-> Ward U is third to the right of Q
-> Ward U is not opposite either C or F.
-> Ward S and T are not opposite C. Ward P is adjacent to ward S
-> Ward D, which is not opposite T, is not adjacent to A

6. Which ward is opposite Q?


1) E
2) D
3) B
4) A
5) None of these
Answer

Answer – 3) B

7. How many wards are there between wards R and S?


1) Four
2) Three
3) Two
4) One
5) None of these

Answer

Answer – 3) Two

8. Ward R is related to ward E in the same way as ward U is


related to B, based on the given arrangement. Who among
the following is Q related to following the same pattern?
1) A
2) B
3) C
4) D
5) None of these

Answer

Answer – 1) A

9. Which of the following wards are at the ends of the building?


1) C, B
2) U, Q
3) C, D
4) R, T
5) None of these

Answer

Answer – 4) R, T
10. Which of the following statements is true about P ?
1) Ward P is 2nd from the right end
2) Ward S is not adjacent to ward P
3) Ward A is not the neighbour of that ward which is opposite P
4) Ward F is opposite ward P
5) None of these

Answer

Answer – 4) Ward F is opposite ward P

Direction(1-5): Study the following information carefully to answer


the given questions
A committee has nine members P, Q, R, S, T, U, V, W and X and it is divided
into three groups, viz Shubhas group, Nehru group and Azad group – with
three members in each group. There are certain conditions for the formation
of the groups. Only three members will be selected in each group. One
person cannot be selected for more than one group. P is in the Shubhas
group. W is in the Nehru group. R and S are in the same group but not in the
group of X. P and Q are not in the same group. T is not in the same group as
X but is in the Azad group. U is in the group in which either W or Q or both of
them are present. X and W are not in the same group. U is neither with T nor
with X.

1. R and S are in which of the following groups?


a) Shubhas group
b) Nehru group
c) Azad group
d) Can’t be determined
e) None of these

Answer
Answer – c) Azad group

2. Which of the following statements is true about V?


a) V is in the Shubhas group with W
b) V is in the Azad group with P
c) V is in the Shubhas group with R and S
d) V is in the Shubhas group with P
e) None of these

Answer

Answer – d) V is in the Shubhas group with P

3. U is associated with which of the following groups?


a) Azad
b) Nehru
c) Shubhas
d) can’t be determined
e) None of these

Answer

Answer – b) Nehru

4. Which of the following members represents the Azad group?


a) RSP
b) TPQ
c) TRU
d) STR
e) None of these

Answer

Answer – d) STR

5. Which of the following combinations is true ?


a) Shubhas group-PQT
b) Azad group – TRQ
c) Nehru group – WUQ
d) Azad group – PXV
e) None of these

Answer

Answer – c) Nehru group – WUQ

P SHUBHAS
Q NEHRU
R AZAD
S AZAD
T AZAD
U NEHRU
V SHUBHAS
W NEHRU
X SHUBHAS

Direction(6-10): Study the following information carefully to answer


the given questions
Seven friends P, Q, R, S, M, N and X go to market on different days –
Sunday, Monday, Tuesday, Wednesday, Thursday, Friday and Saturday for
part-time jobs at different places, viz Jaipur, Udaipur, Ajmer, Delhi, Noida,
Bhopal and Indore but not necessarily in the same order. R works at the
Ajmer but goes there neither on Saturday nor on Thursday. M goes on
Tuesday and works at Bhopal. P goes on Friday but works neither at Jaipur
nor at Noida. The one who goes on Monday works at Udaipur. The one who
works at Jaipur goes there on Wednesday. N works at Noida. X works at
Delhi and goes on Saturday. S does not go on Monday

6. Who among the following works at Udaipur?


a) P
b) Q
c) R
d) X
e) None of these
Answer

Answer – b) Q

7. On which day does N go to market?


a) Monday
b) Tuesday
c) Wednesday
d) Thursday
e) None of these

Answer

Answer – d) Thursday

8. Which of the following combinations is true?


a) M – Thursday – Bhopal
b) S – Tuesday – Jaipur
c) R – Sunday – Ajmer
d) N – Friday – Noida
e) None of these

Answer

Answer – c) R – Sunday – Ajmer

9. Who among the following goes to work on Sunday?


a) Q
b) N
c) M
d) P
e) None of these

Answer

Answer – e) None of these

10. The person going on Saturday works in which of the


following place?
a) Jaipur
b) Bhopal
c) Delhi
d) Indore
e) None of these

Answer

Answer – c) Delhi

P FRIDAY INDORE

Q MONDAY UDAIPUR

R SUNDAY AJMER

S WEDNESDAY JAIPUR

M TUESDAY BHOPAL

N THRUSDAY NOIDA

X SATURDAY DELHI

Direction(1-5): Study the following information carefully to answer


the given questions
Six persons P, Q, R, S, T and U are sitting around a circular table facing
towards the centre of the table in a restaurant. They have ordered for
different items i.e. samosha, cutlet, French fries, omelette, sandwich and
milk-cake. They are wearing T-shirts of different colours, i.e. white, black,
green, red, yellow and blue but not necessarily in the same order.

 The person who have ordered for samosha, French fries and
Sandwich are neither in white nor in black T- shirt.
 The person who are in green and yellow T-shirts have neither
ordered for samosha nor for French fries
 P is neither in white T-shirt nor on the immediate left of the person
who has ordered for omelette.
 The only person who is seated between T and U eats cutlet. The
person who is on the left side of the person in white T-shirt does not
eat Milk-cake.
 S has ordered for omelette and the colour of his T-shirt is green. He
is facing the person who has ordered for Cutlet.
 One who has ordered for samosha is seated opposite to the person
wearing blue T-shirt, while the person whose T-shirt is of green
colour is on the left of the person who has ordered for sandwich.
 One who has ordered for milk-cake is on the immediate right of the
person in white T-shirt but on the immediate left of the person who
has ordered for French fries
 R has not ordered for french-fries while U has not ordered for
samosha

1. Who among the following is in white T-shirt?


a) P
b) T
c) U
d) R
e) None of these

Answer

Answer – d) R

2. The only person between T and S is wearing T –shirt of which


colour?
a) red
b) black
c) yellow
d) green
e) None of these
Answer

Answer – c) yellow

3. Who among the following has ordered for sandwich?


a) R
b) S
c) P
d) T
e) None of these

Answer

Answer – c) P

4. Which of the following is correctly matched?


a) R-WHITE-MILKCAKE
b) U-BLACK-SAMOSHA
c) T-RED-SAMOSHA
d) P-WHITE-OMELETE
e) None of these

Answer

Answer – c) T-RED-SAMOSHA

5. The colour of the T-shirt of the person, who has ordered for
milk-cake, is-
a) green
b) yellow
c) blue
d) red
e) None of these

Answer

Answer – e) None of these


Direction(6-10): Study the following information carefully to answer
the given questions
There are six boys P, Q, R, S, T and U. They want to go out with six girls A,
B, C, D, E and F, not necessarily in the same order. The pairs went to
different cities Delhi, Mumbai, Kolkata, Chennai and two of them went to
Jaipur. They like different bikes viz KTM, Bullet, Pulsar and Apache. But KTM
and Bullet are preferred by two pairs. Further information is as follows:

 P and S visit Jaipur but do not like either KTM or Apache


 U does not go out with E. Both of them do not like Pulsar.
 D and C wants to go to Delhi and Kolkata respectively. The persons
visiting Mumbai like the same bike as Q does.
 R goes out with F to Mumbai but does not like either Bullet or
Apache.
 Q goes to Delhi and likes KTM.
 T does not go out either with A or B; he does not go to Kolkata; he
likes neither Pulsar nor Bullet.

6. Which bike does C like?


a) KTM
b) Bullet
c) Apache
d) Pulsar
e) None of these

Answer

Answer – b) Bullet

7. If P goes out with A, who among the following likes Pulsar ?


a) Q
b) R
c) T
d) P
e) can’t be determine

Answer
Answer – e) can’t be determine

8. Who went to Chennai?


a) RF
b) UC
c) QD
d) TE
e) None of these

Answer

Answer – d) TE

9. Which bike does C like?


a) Apache
b) Pulsar
c) Bullet
d) KTM
e) None of these

Answer

Answer – c) Bullet

10. Who among the following visits Kolkata?


a) C
b) Q
c) P
d) U
e) Both a and d

Answer

Answer – e) Both a and d


Direction(1-5): Study the following information carefully to answer
the given questions

There are seven Indian footballers, namely A, B, C, D, E, F and G. Each of


them belongs to a different state like Maharashtra, Jharkhand, UP, Punjab,
Delhi, Karnataka and Tamil Nadu. Moreover, each of them also has a liking
for different foods, namely Chicken Tikka, Veg. Biryani, Fish Curry, Chicken
Biryani, Mutter Paneer, Mutton Curry and Afghani Chicken but not necessarily
in the same order.
A, who likes Fish Curry, is neither from Maharashtra nor from Punjab.
E is not from Jharkhand and does not like Chicken Tikka. F, who hails from
Tamil Nadu, does not like Veg. Biryani.
D, who likes Afghani Chicken, is not from Karnataka. The one who is from
Jharkhand is fond of Mutton Curry.
C likes Mutter Paneer and is from UP while B, who likes Veg. Biryani is from
Delhi

1. Who among the following belong to Karnataka?


a) A
b) B
c) C
d) D
e) None of these

Answer

Answer – a) A

2. D belongs to which of the following states?


a) Jharkhand
b) UP
c) Delhi
d) Tamil Nadu
e) can’t be determined

Answer
Answer – e) can’t be determined

3. Which of the following statements is/are false?


a) A belongs to Karnataka and likes Fish Curry.
b) G likes Mutton Curry.
c) C is from Jharkhand.
d) None is true
e) All are True

Answer

Answer – c) C is from Jharkhand.

4. Who among the following belongs to Punjab?


a) G
b) E
c) D
d) None of these
e) can’t be determined

Answer

Answer – e) can’t be determined

5. Chicken Tikka is the favourite food of which among the


following?
a) F
b) E
c) G
d) A

Answer

Answer – a) F

A KARNATAKA FISH CURRY

B DELHI VEG BIRYANI


C UP MUTTER PANEER

D MAHARASHTRA/PUNJAB AFGHANI CHICHKEN

E MAHARASHTRA/PUNJAB CHICKEN BIRYANI

F TAMIL NADU CHICKEN TIKKA

G JHARKHAND MUTTON CURRY

Direction(6-10): Study the following information carefully to answer


the given questions

Eight persons from different companies, like Hero, Honda, Bajaj, TVS, Harley
Davidson, Royal Enfield, KTM and Suzuki are sitting in two parallel rows
containing four persons each in such a way that there is an equal distance
between adjacent persons. In row 1- P, Q, R and S are sitting and all of them
are facing north. In row – 2, A, B, C and D are sitting and all of them are
facing south- Therefore, in the given seating arrangement each member
sitting in a row faces another member of the other row. All the information
given above does not necessarily represent the; order of seating, as in the
final arrangement.

 The person from Harley Davidson faces the one who sits on the
immediate left of R. R is neither from Honda nor from Bajaj.
 An immediate neighbour of A faces the person from Hero. The
person from Royal Enfield faces the person who sits on the left of
the person from Honda.
 There is only one person sitting between the persons from Honda
and KTM but that person is not R. The persons from Honda and
Bajaj are not sitting at the extreme ends. B sits on the immediate
left of the person from Harley Davidson. The persons from TVS and
Royal Enfield are immediate neighbours. C and Q are not sitting at
any of the ends.
 S faces the one who is sitting on the immediate right of the person
from Royal Enfield. A is not from Royal Enfield or TVS. P does not
face Royal Enfield

6. Who among the following is from Suzuki?


a) A
b) R
c) B
d) S
e) None of these

Answer

Answer – c) B

7. Who is sitting between D and the person from Harley


Davidson?
a) The person who is from Bajaj
b) B
c) S
d) The person who faces the one who is from Bajaj
e) None of these

Answer

Answer – d) The person who faces the one who is from Bajaj

8. R is from which of the following companies?


a) Hero
b) Bajaj
c) Honda
d) TVS
e) None of these

Answer

Answer – a) Hero
9. Who among the following faces the one who is from Royal
Enfield?
a) The person who is from Mahindra
b) R
c) S
d) Q
e) None of these

Answer

Answer – d) Q

10. How many persons are sitting between the S and the
person who is from Bajaj?
a) none
b) one
c) two
d) three
e) M=more than Four

Answer

Answer – a) none

Direction(1-6): Study the following information carefully to answer


the given questions

A, B, C, D, E, F, G and H are eight persons. Each is a professor of a different


subject like Maths, Physics, Hindi, English, Geology, Zoology, Botany and
History. Each of them having different hobby like Riding, Music, Poetry,
Shayari, Painting, Rafting, Cooking and Swimming but not necessarily in the
same order. A who likes cooking is a professor of neither History nor
Geology. B is a professor of English and does not like either Riding or Poetry.
The one who is a professor of Maths likes Shayari. The one who likes
Swimming is a professor of Physics and the one who is a professor of Botany
likes Music. H doesn’t like Painting. E and F like Music and Rafting, though
not necessarily in the same order. D likes Swimming. G is a professor of
Zoology and doesn’t like Riding. The one who is a professor of Geology likes
Rafting. C is not a professor of History. The one who is a professor of Hindi
doesn’t like either Poetry or Painting.

1. Who among of the following likes Riding?


a) A
b) B
c) D
d) H
e) None of these

Answer

Answer –d) H

2. Who among of the following is a professor of History?


a) E
b) F
c) B
d) H
e) None of these

Answer

Answer – d) H

3. C is a professor of which of the following subjects?


a) Maths
b) Geology
c) Zoology
d) Hindi
e) None of these

Answer
Answer – a) Maths

4. Which of the following combinations is definitely correct?


a) C-Maths-Music
b) F-Botany-Music
c) D-Zoology-Riding
d) B-English-Painting
e) None is correct

Answer & Explanation

Answer -d) B-English-Painting


Explanation :

5. Which of the following combinations is incorrect?


a) D-Swimming
b) A-Hindi
c) G-Rafting
d) C-Shayari
e) None of these

Answer

Answer – c) G-Rafting

6. Which of the following is G’s hobby?


a) Painting
b) Cooking
c) Riding
d) Poetry
e) None of these

Answer

Answer – d) Poetry

A HINDI COOKING
B ENGLISH PAINTING
C MATH SHAYARI
D PHYSICS SWIMMING
E GEOLOGY/BOTANY MUSIC/RAFTING
F GEOLOGY/BOTANY MUSIC/RAFTING
G ZOOLOGY POETRY
H HISTORY RIDING

Direction(7-10): Study the following information carefully to answer


the given questions

Seven participants P, Q, R, S, T, U and W are sitting in a straight line facing


east. All of them have participated in a hundred-metre race. Q sits fourth to
the right of the person whose rank is 6th in the race. Either Q or the sixth
ranker sits at the extreme end of the line. Only one person sits between Q
and W. W is the third ranker. The person whose rank is first sits third to the
left of S. S is not an immediate neighbour of W. Only one person sits
between T and the person who finished at the second place. P and R are
immediate neighbours. P is not at the sixth rank. The person whose rank is
fifth sits third to the left of the person who finished seventh

7. Who ranked at the fourth place?


a) R
b) S
c) W
d) T
e) None of these

Answer

Answer – b) S

8. T’s rank in the race is-


a) second
b) third
c) fourth
d) first
e) None of these

Answer

Answer – d) first

9. How many persons are sitting between P and U?


a) one
b) two
c) three
d) four
e) None of these

Answer & Explanation

Answer -c) three


Explanation :

10. Who is sitting third to the right of the person who got
5th rank?
a) R
b) P
c) Q
d) S
e) None of these

Answer

Answer – c) Q

Direction(1-5): Study the following information carefully to answer


the given questions

a) A, B, C, D, E, F, G and H are eight students in the class. Three of them


study English and Commerce each and two of them study Sanskrit. Each one
of them has a different weight.
b) The heaviest does not study Sanskrit and the lightest does not study
English.
c) F is heavier than A and D, but lighter than H and B. E, who does not study
English, is heavier than B and is the second heaviest. G is lighter than D but
heavier than A.
d) H, who is fourth from the top, studies Commerce along with D.
e) G does not study either English or Sanskrit. B does not study Sanskrit

1. Who is the heaviest?


a) E
b) B
c) C
d) D
e) none of these

Answer

Answer – c) C

2. Who among the following is lighter than D?


a) C
b) G
c) E
d) F
e) none of these

Answer

Answer – b) G

3. Which of the following pairs studies Sanskrit?


a) FG
b) AC
c) CE
d) BE
e) AE

Answer
Answer – e) AE

4. Which of the following groups of students studies English?


a) ABF
b) BCE
c) ABC
d) BCA
e) BCF

Answer

Answer – e) BCF

5. Who is the lightest person in the students?


a) F
b) D
c) C
d) A
e) none of these

Answer

Answer – d) A

A SANSKRIT
B ENGLISH
C ENGLISH
D COMMERCE
E SANSKRIT
F ENGLISH
G COMMERCE
H COMMERECE

Direction(6-10): Study the following information carefully to answer


the given questions
There is a group of six persons living in a four-storey building. The persons
are P, Q, R, S, T and U. Each storey of the building has two flats. Thus, there
are eight flats in all and two of them are vacant.
a) R is heavier than Q, who in turn is heavier than U.
b) The heaviest of the group lives on the top floor while the lightest lives on
the ground floor.
c) T is lighter than S.
d) One of the flats on the first floor is vacant while one on the second floor is
also vacant.
e) P is neither the heaviest nor the lightest.
f) There are only two people heavier than R. P is not one of them.
g) Q shares the floor with the heaviest of the group while R shares it with the
lightest

6. Who among the following is the lightest in the group?


a) P
b) R
c) U
d) Q
e) None of these

Answer

Answer – c) U

7. On which floor does P live?


a) ground
b) first
c) second
d) either first or second

Answer

Answer – d) either first or second

8. Who among the following shares the floor with R?


a) Q
b) P
c) S
d) U
e) None of these

Answer

Answer – d) U

9. T and P
a) Lives on the same floor
b) Have two floors between their floors
c) Lives on adjacent floors
d) Have one floor between them
e) None of these

Answer

Answer – c) Lives on adjacent floors

10. Starting from the heaviest of the group, what is the rank
of Q in order of weight?
a) 3rd
b) 4th
c) 5th
d) either 4th or 5th

Answer

Answer – d) either 4th or 5th

Direction(1-5): Study the following information carefully to answer


the given questions
Amongst five friends, A, B, C, D and E, each got a different percentage of
marks in the class 12th examination, D scored more than B but less than E. B
scored 80% marks. The one who scored the minimum marks, scored 75%
marks, and the one who scored the highest, scored 97% marks. C scored
more than only A

1. Who scored the second lowest marks?


a) C
b) A
c) B
d) E
e) None of these

Answer

Answer – a) C

2. Who among the following is most likely to have scored 85%


marks?
a) C
b) D
c) E
d) Can’t be determined
e) None of these

Answer

Answer – b) D

3. Which of the following could possibly be C’s percentage?


a) 82%
b) 80%
c) 75%
d) Can’t be determined
e) None of these

Answer
Answer – e) None of these

4. Which of the following is true with respect to the given


information?
a) C’s percentage was definitely less than 65%.
b) E scored the second highest percentage.
c) Only two people scored more than A.
d) The possible percentage obtained by D is 98%.
e) None of these

Answer

Answer – e) None of these

5. Which of the following is false with respect to the given


information?
a) B scored more than only C and A
b) C scored 80% marks
c) E scored the highest percentage.
d) A scored the least percentage.
e) All are true

Answer

Answer – b) C scored 80% marks

Direction(6-10): Study the following information carefully to answer


the given questions

P,Q, R, S, A, B, C and D are eight friends studying in engineering colleges in


different streams like Aeronautical Engineering, Computer Engineering,
Chemical Engineering, Electronic Engineering, Electrical Engineering, Civil
Engineering, Mechanical Engineering and Technical Engineering, but not in
the same order. All of them are seated around a circular table, facing away
from the centre. P sits third to the left of the person who studies Electrical
Engineering. Only two persons sit between A and C. Neither A nor C studies
Electrical Engineering and neither of them is an immediate neighbor of P. The
person who studies Technical Engineering sits second to the right of S. S is
not an immediate neighbour of P. S does not study Electrical Engineering and
P does not study Technical Engineering. The one who studies Aeronautical
Engineering sits third to the left of B. The one who studies Aeronautical
Engineering and the one who studies Electrical Engineering are not
immediate neighbours. Only one person sits between S and the one who
studies Chemical Engineering. The person who studies Electronic Engineering
and the one who studies Computer Engineering are immediate neighbours.
Only one person sits between Q and the one who studies Civil Engineering.
The one who studies Civil Engineering is an immediate neighbour of D. C and
Q are not immediate neighbours. S does not study Computer Engineer

6. How many people sit between A and the one who studies
Civil Engineering (when counted in clockwise direction from
A)?
a) one
b) two
c) three
d) four
e) five

Answer

Answer – d) four

7. Who studies Chemical Engineering?


a) R
b) B
c) Q
d) P
e) A

Answer

Answer – e) A

8. Which of the following combinations is, definitely true?


a) S – Electrical Engineering
b) R-Civil Engineering
c) C-Chemical Engineering
d) Q – Electronic Engineering
e) None of these

Answer

Answer – b) R-Civil Engineering

9. Who studies Aeronautical Engineering?


a) P
b) A
c) Q
d) R
e) S

Answer

Answer – a) P

10. What is the position of the person who studies Mechanical


Engineering with respect to R?
a) Immediate left
b) Second to the left
c) Third to the right
d) Third to the left
e) Second to the right

Answer

Answer – e) Second to the right

Direction(1-5): Study the following information carefully to answer


the given questions
A, B, C, D, E and F are six members of a group in which there are three
female members. Females work in three departments –Maintenance,
Admission and HR – and sit on three different floors – Ground floor, first floor
and second floor. Person working in the same department are not on the
same floor. On each floor two persons work. No two females work in the
same department or sit on the same floor. B and E work in the same
department but not in HR. D works in Admission. E and A are on the ground
floor and second floor respectively and work in the same department. D, a
female does not sit on the first floor. C, a male works on ground floor. F,
works on the first floor and D and A work on the same floor

1. Which of the following group of members are female?


a) ABC
b) DEF
c) ACF
d) ADE
e) Can’t be determined

Answer

Answer – b) DEF

2. Which of the following pair of person work in Admission?


a) AC
b) CD
c) DA
d) Data inadequate
e) none of these

Answer

Answer – d) Data inadequate

3. Which of the following pair works on second floor?


a) AE
b) AF
c) AD
d) DC
e) None of these

Answer

Answer – c) AD

4. F works in which department?


a) Maintenance
b) HR
c) Admission
d) Either HR or Admission
e) None of these

Answer

Answer – b) HR

5. Which of the following represents people from all three


floors?
a) FBA
b) CFB
c) BEC
d) ADB
e) AFC

Answer

Answer – e) AFC

A Maintenance MALE 2nd floor


B Maintenance MALE 1st floor
C HR/Admission MALE Ground floor
D Admission FEMALE 2nd floor
E Maintenance FEMALE Ground floor
F HR FEMALE 1st floor
Direction(6-10): Study the following information carefully to answer
the given questions

One of the seven Hollywood films—Titanic, Inception, Shutter island,


Revenant, X-men, Batman and Ghost rider—is shown on each day of a Film
Festival, starting from Monday and ending on Sunday. Batman film is shown
on Thursday. X-men film is shown on the day immediately after the day on
which Inception film is shown. X-men film is shown neither on Tuesday nor
on Saturday. Only one film is shown between Batman and Shutter Island
film. Two films are shown between Titanic and Inception film. Ghost rider film
is shown neither on Sunday nor on Monday

6. Revenant film is shown on which of the following days?


a) Monday
b) Tuesday
c) Wednesday
d) Thursday
e) Friday

Answer

Answer – a) Monday

7. How many films are shown between Shutter Island and


Inception?
a) One
b) Two
c) Three
d) Four
e) None of these

Answer

Answer – c) Three

8. Which of the following movies is shown on Saturday?


a) Ghost rider
b) Shutter Island
c) Titanic
d) Inception
e) None of these

Answer

Answer – d) Inception

9. On which of the following days is Ghost Rider shown?


a) Tuesday
b) Wednesday
c) Thursday
d) Friday
e) Cannot be determined

Answer

Answer – d) Friday

10. Which of the following movies is shown just before the


Batman film?
a) Ghost Rider
b) Titanic
c) Shutter Island
d) Inception
e) Cannot be determined

Answer

Answer – b) Titanic

Titanic WEDNESDAY
Inception SATURDAY
Shutter Island TUESDAY
Revenant MONDAY
X –Men SUNDAY
Batman THURSDAY
Ghost rider FRIDAY
irection(1-5): Study the following information carefully to answer the
given questions

Seven actors A, B, C, D, E, F and G act in different types of drama like


Action, Thriller, Romantic, Suspense, Comedy, Horror and Inspirational, but
not necessarily in the same order. All the seven actors are engaged on three
different days of the week i.e. Sunday, Monday and Tuesday. A is engaged
on Sunday only with E and his drama is Thriller. D is acting in Horror drama
and does not act on Tuesday. The one who is acting in Comedy acts on
Monday. B and C do not act on the same day. Those who act on Sunday do
not perform Romantic drama. F acts in Suspense drama but does not act on
Monday. G acts on the same day as F. C acts in Action drama

1. Who acts in Comedy?


a) A
b) B
c) E
d) F
e) none of these

Answer

Answer – b) B

2. Which of the following groups of actors performs on


Tuesday?
a) ABG
b) CFE
c) CDE
d) CFG
e) EFG

Answer

Answer – d) CFG
3. B and D act on which of the following days of the week?
a) Sunday
b) Monday
c) Tuesday
d) Monday and Tuesday
e) none of these

Answer

Answer – b) Monday

4. E acts in which type of drama?


a) Thriller
b) Action
c) Comedy
d) Horror
e) none of these

Answer

Answer – e) none of these

5. Which of the following combinations of actor-day-drama is


definitely correct?
a) F – Monday – Romantic
b) G – Tuesday – Romantic
c) D – Sunday – Action
d) E – Tuesday – Inspirational
e) None of these

Answer

Answer – b) G – Tuesday – Romantic

A THRILLER SUNDAY
B COMEDY MONDAY
C ACTION TUESDAY
D HORROR MONDAY
E INSPIRATIONAL SUNDAY
F SUSPENSE TUESDAY
G ROMANTIC TUESDAY

Q(6-10): Study the following information to answer the given


Questions

A university organised exams for six different subjects i.e. Hindi, Sanskrit,
Biology, Marketing, Computer and Finance on six days of a week, not
necessarily in the same order. The exams starts from Monday, with a holiday
on any day of the week, one full day is devoted to one exam. The exam of
Hindi is scheduled immediately after the exam of Sanskrit. The exam of
Marketing is scheduled on Wednesday but not after the exam of Finance. The
exam of Biology is scheduled on Friday. There is only one exam between the
exams of Computer and Hindi. There is only one day when no paper is
scheduled but that is not Saturday. The exam of Finance is scheduled just
before the holiday

6. On which of the following days is the exam of Computer


scheduled?
a) Monday
b) Tuesday
c) Friday
d) Saturday
e) none of these

Answer

Answer – e) none of these

7. Which two exams are scheduled on the first and last day?
a) Sanskrit and Biology
b) Biology and Finance
c) Sanskrit and Finance
d) Hindi and Finance
e) none of these
Answer

Answer – c) Sanskrit and Finance

8. On which day is a holiday?


a) Sunday
b) Monday
c) Tuesday
d) Wednesday
e) None of these

Answer

Answer – a) Sunday

9. How many exams is/are scheduled between the exams of


Hindi and Marketing?
a) one
b) two
c) three
d) four
e) none of these

Answer

Answer – e) none of these

10. Which of the following combinations is correct?


a) Finance-Thursday
b) Hindi-Monday
c) Computer-Saturday
d) Sanskrit-Monday
e) None of these

Answer

Answer – d) Sanskrit-Monday
MONDAY SANSKRIT
TUESDAY HINDI
WEDNESDAY MARKETING
THURSDAY COMPUTER
FRIDAY BIOLOGY
SATURDAY FINANCE
SUNDAY HOLIDAY

I.Study the following information carefully and answer the questions


given below

There are eight persons in a family M, N, O , P, Q, R, S and T are


sitting around a circular table facing the centre. Among them there are three
doctors, two Engineers, two Authors and one Painter. Each person has only
one occupation. There are three married couples in the family and no
unmarried female. Three generation are there in the family

P has two children. S and T are children of Q and O respectively. R is wife of


P and mother of N and M. O is a female but not wife of M

S, a grandson of P, sits exactly between O and M and only Q sits exactly


between N and R. Only one person sits between O and N but he is not T. N is
not the immediate neighbour of S, who is not opposite to P. N does not sit
opposite to a female. One of the sons of P is on immediate left of P. The
person who is on the immediate left of S and the person who is on the
immediate right of N are not doctors. The immediate neighbours of Q are
neither an Engineer nor a Doctor. T is a Painter

1. Who among the following sits exactly between P and Q ?


1.Author
2.Doctor
3.Engineer
4.Painter
5.None of these
Answer

1.Author

2. Which of the following does not form a group ?


1.R
2.M
3.Q
4.O
5.None of these

Answer

2.M

3. Which of the following two members sits adjacent to each


other in the arrangement ?
1.QO
2.NT
3.TQ
4.SM
5.None of these

Answer

4.SM

4. Who among the following is Son of Q ?


1.T
2.O
3.S
4.P
5.None of these

Answer

3.S
5. Who among the following is an Engineer ?
1.P
2.O
3.T
4.Both 1 and 3
5.Both 1 and 2

Answer

5.Both 1 and 2

II .Study the following information carefully and answer the


questions given below

A, B, C, D, E, F, G and H are sitting around a square table in such a


way that four of them sits at four corners of the square facing the centre
while four others sit in the middle of each of the four sides but not facing the
centre. A sits third to the right of F, who is not facing the centre.

 E is not an immediate neighbour of F or A. There is only one person


between F and G.
 D sits second to the right of B, who is on the immediate left of G
 C is not an immediate neighbour of A.

6. Who among the following sits opposite to F ?


1.E
2.C
3.H
4.A
5.None of these

Answer

3.H

7. Who among the following sits exactly between G and H ?


1.A
2.B
3.D
4.F
5.None of these

Answer

1.A

8. Four of the following five are alike in a certain way and


hence form a group.Which of the following does not belong
to that group ?
1.A
2.B
3.D
4.G
5.None of these

Answer

4.G

9. What is the position of B with respect to D ?


1.Third to the right
2.Second to the left
3.Immediate right
4.Second to the right
5.None of these

Answer

2.Second to the left

10. If E is facing outside then which will be the immediate


right of E ?
1.H
2.F
3.G
4.D
5.None of these
Answer

4.D

irection(1-5); Study the following information carefully to answer the


given questions

A, B, C, D, E, F, G and H live on eight different floors of a building but not


necessarily in the same order. The lowermost floor of the building in
numbered 1, the above one that is numbered 2 and so on/ Each of them
likes a different fruit, namely Banana, Strawberry, Apple, Turnip, Guava,
Kiwi, Mango and Grapes but not necessarily in the same order.

 The one who likes Turnip lives on an even numbered floor. Only
three people live between who likes Turnip and C.Only two people
live between C and D. D does not live on the lowermost floor. Only 3
people live between D and the one who likes Guava.
 E lives immediately above A. E lives on an even numbered floor. A
does not like Guava. A lives neither on the floor numbered 3 or
5.Only two people live between A and the one who likes Mango.
Only one person lives between the one who likes Mango and
Grapes. The One who likes Grapes below the one who likes
Strawberry.
 B lives immediately above G. Only one person lives between G and
the one who likes Apple. The one who likes Kiwi lives immediately
above the one who likes Banana. F does not like Turnip and A does
not like Strawberry.

1. Who lives between H and one who likes Mango ?


1.D
2.F
3.A
4.G
5.None of these

Answer

Answer – 1.D

2. B lives on which of the following floor number ?


1.7
2.5
3.4
4.6
5.None of these

Answer

Answer – 3.4

3. Which of the following Fruit does A likes ?


1.Banana
2.Strawberry
3.Guava
4.Mango
5.None of these

Answer

Answer – 1.Banana

4. Which of the following is correct ?


1.A-8-Apple
2.C-2-Grapes
3.D-6-Guava
4.E- 4-Turnip
5.None of these

Answer

Answer – 2.C–2-Grapes
5. The Favourite Fruit of H is ……………………..
1.Apple
2.Kiwi
3.Turnip
4.Grapes
5.None of these

Answer

Answer – 3.Turnip

Floor Person Fruit


8 E Kiwi
7 A Banana
6 H Turnip
5 D Apple
4 B Mango
3 G Strawberry
2 C Grapes
1 F Guava

Direction(6-10): Study the following information carefully to answer


the given questions

Ten Persons X, Y, Z, H, I, U, V, W, G and E are sitting in two rows


with five persons in each row. The persons in row one are facing South and
the persons in row two are facing North. Each person in row one faces a
person from other row.

 All of them likes a different soft drink – Sprite, Slice, Pepsi, Maaza,
Fanta, Mirinda, Limca, 7up, Nimbooz and Cola but not necessarily in
the same order.
 The persons who likes Fanta and Mirinda sit opposite each other. U
sits opposite X, who likes Sprite. The one who likes Slice sits
opposite the one who likes 7up. E is not facing north but sits third to
the left of V, Who likes Slice. There is only one person between Y
and Z. I sits at one of the ends of the row and likes Mirinda. The one
who likes 7up is on the immediate right of H, who does not like
Cola. The person who likes Pepsi and Maaza respectively are not
facing north. Z likes Limca.The one who likes Maaza sits opposite
the one who is second to the right of Y. G does not like Cola. I sits
opposite the one who sits second to the left of one who likes Pepsi

6. How many persons sits between H and I ?


1.1
2.4
3.2
4.3
5.None of these

Answer

Answer – 4.3

7. Y likes which of the following soft drink ?


1.Maaza
2.Sprite
3.7up
4.Cola
5.Fanta

Answer

Answer – 3.7up

8. Who among the following sit at the extreme end of the row
and are facing south ?
1.W, E
2.U, V
3.V, W
4.H, I
5.None of these

Answer
Answer – 1.W, E

9. Four of the following five alike in a certain way and hence


form a group, which of the one does not belong to that group
?
1.W, H
2.V, Y
3.E, I
4.G, Y
5.U, X

Answer

Answer – 4.G, Y

10. Who among the following sits second to the right of one
who likes Maaza ?
1.U
2.V
3.W
4.I
5.None of these

Answer

Answer – Ans: 2.V

I.Study the following information carefully to answer the given


questions

Seven employees A, B, C, D, E, F and G working in different companies reach


their office by a train which stops at 5 stations P, Q, R,S and T respectively
after leaving station X.
 Three of them board the train at Station X. D deboards at the
station next to the station at which F deboards. B does not deboards
either with A or with E. G alone boards at station R and deboards
with C after passing one station.
 None of them boards at station Q and none of them deboards the
train at station P. C boards with F but does not board with either B
or with D.
 E boards with two others and deboards alone after D. B and D
deboard together at station R.A travels between two consecutive
stations and deboards at the station T. F does not board after E

1. Who among the following board at Station P ?


1.B
2.D
3.C
4.Both 1 and 2
5.Both 1 and 3

Answer

Answer – 4.Both 1 and 2

2. A boards the train how many station after F deboards ?


1.One
2.Two
3.Three
4.Four
5.None of these

Answer

Answer – 1.One

3. After how many station does C deboard ?


1.One
2.Two
3.Three
4.Four
5.None of these

Answer

Answer – 4.Four

4. At which of the following station does E deboard ?


1.P
2.S
3.Q
4.R
5.None of these

Answer

2.S

5. At which of the following station do C and F board the train ?


1.R
2.P
3.X
4.T
5.None of these

Answer

Answer – 3.X

Station Gets in Gets down


X E, F, C –
P B, D –
Q – F
R G B, D
S A E
T – A, C, G
II.Study the following information carefully to answer the given
questions

A, B, C, D, E, F, G and H live on eight different floors of a building but not


necessarily in the same order. The ground floor is numbered one and the
floor above it is numbered two and so on. The top most floor is numbered
eight. Each of them likes a different cities viz Agra, Mumbai, Delhi, Kolkata,
Cochin, Pune, Kashmir and Chennai but not necessarily in the same order.

 The One who likes Mumbai lives on an even numbered floor but not
on floor no.8. Only two persons live between F and the one who
likes Mumbai. Only one person live between F and the one who likes
Cochin. Neither E nor C lives on floor no 1. Only one person lives
between C and D,who likes Delhi. A lives immediately above F, who
lives on an odd numbered floor.
 Only two persons live between E and A. The one who likes Cochin
does not live on floor no.1. B lives on an even numbered floor and
immediately above C. The one who likes Kolkata lives on an even
numbered floor and live immediately above the person who likes
Chennai. C does not like Cochin or Chennai. Only two persons live
between H and the one who likes Agra. The one who likes Pune does
not live on odd numbered floor. H does not like Kashmir. B lives on
floor no.4

6. Who among the following live on floor 5 ?


1.A
2.C
3.E
4.G
5.None of these

Answer

Answer – 3.E

7. B likes which of the following city ?


1.Agra
2.Chennai
3.Kolkata
4.Mumbai
5.None of these

Answer

Answer – 4.Mumbai

8. Who among the following likes Pune ?


1.F
2.H
3.D
4.B
5.None of these

Answer

Answer – 2.H

9. Who lives between A and H ?


1.E
2.B
3.C
4.F
5.None of these

Answer

Answer – 4.F

10. Which of the following is not correct ?


1.4-B-Mumbai
2.7-F-Chennai
3.5-E-Cochin
4.3-H-Kashmir
5.None of these

Answer
Answer – 4.3-H-Kashmir

Floor Person City


8 A Kolkata
7 F Chennai
6 H Pune
5 E Cochin
4 B Mumbai
3 C Agra
2 G Kashmir
1 D Delhi

I.Study the following information carefully to answer the given


questions

M, N, O, P, Q, R and S are employees of a company. Each of them


works in his own cabin numbered 1 to 7, but not necessarily in the same
order. Each of them wears a shirt of different colours – Blue, Green , Yellow,
Sky Blue, Purple, Red and Pink but not necessarily in the same order.

N sits in cabin 4 but he does not wear either purple or sky blue shirt. O
wears Blue shirt but he does not sit in cabin 2 or 6. Q sits in cabin 5 and he
wears a Red shirt. The one who wears a Green shirt sits in cabin 7, P sits in
cabin 1. S wears a Pink shirt. M does not sit in cabin 7. The one who wears
Sky blue shirt sits in cabin 2.Either M or R does not sit in cabin 6

1. M wears a shirt of which of the following colours ?


1.Green
2.Blue
3.Sky Blue
4.Pink
5.None of these

Answer

Answer – 3.Sky Blue

2. S sits in which of the following Cabin ?


1.5
2.3
3.7
4.6
5.None of these

Answer

Answer – 4.6

3. Who among the following sits in Cabin 2 ?


1.O
2.M
3.Q
4.R
5.None of these

Answer

Answer – 2.M

4. Who among the following wearing blue colour shirt ?


1.O
2.N
3.R
4.T
5.None of these

Answer

Answer – 1.O
5. Which of the following is true ?
1.R-Pink-6
2.O-Red-4
3.1- Red – Q
4.N-Yellow-4
5.None of these

Answer

Answer – 4.N-Yellow-4

Person Colour Cabin


M Sky Blue 2
N Yellow 4
O Blue 3
P Purple 1
Q Red 5
R Green 7
S Pink 6

II.Study the following information carefully to answer the given


questions

Six players Arun, Kamal, Ram, Varun, Yuvraj and Ajay are playing
different types of Sports viz Carom, Chess, Kho-Kho, Tennis, Hockey and
Badminton, but not necessarily in the same order. All are wears a T-shirt of
different colour – Violet, Green, Blue, Orange, Red and yellow.They use
different types of mobiles viz Samsung, Lenovo and Nokia but not necessarily
in the same order.Atleast two persons use the same same types of mobiles.

The person who plays Hockey uses Nokia. Ajay uses Nokia and wears
Green T-Shirt, but he plays neither Carom nor Kho-Kho. The one plays
Tennis uses Samsung. Kamal uses the same types of mobile phone as the
person who wears a Violet T-Shirt, but he is not a chess Player. The one who
is player of Carom uses Samsung. Varun is a Hockey Player and he wears
neither Blue T-shirt nor Yellow T-shirt. Lenovo is used by a chess player,
Yuvraj is a tennis player and wears an orange T-Shirt, but he does not use
the same mobile phone as Ram. Arun does not wear a Red or Yellow shirt.
The players who use Nokia play neither Tennis nor Kho-Kho. The players who
wear Orange and Blue T-shirts use the same type of mobile

6. Yuvraj uses the which of the following mobiles ?


1.Samsung
2.Nokia
3.Lenovo
4.Either Nokia or Lenovo
5.None of these

Answer

Answer – 1.Samsung

7. Who among the following is a Badminton Player ?


1.Arun
2.Ram
3.Yuvraj
4.Ajay
5.None of these

Answer

Answer – 4.Ajay

8. Kamal wears which colour T-Shirt ?


1.Violet
2.Red
3.Yellow
4.Orange
5.None of these

Answer

Answer – 3.Yellow
9. Ram is a ………….. player
1.Kho-Kho
2.Chess
3.Hockey
4.Carrom
5.None of these

Answer

Answer – 2.Chess

10. Which of the following is not true ?


1.Ajay-Blue-Lenovo
2.Arun-Carrom-Samsung
3.Yuvraj-Tennis-Orange
4.Varun-Red-Nokia
5.None of these

Answer

Answer – 1.Ajay-Blue-Lenovo

Person Game T-Shirt Mobile


Arun Carrom Blue Samsung
Kamal Kho-Kho Yellow Lenovo
Ram Chess Violet Lenovo
Varun Hockey Red Nokia
Yuvraj Tennis Orange Samsung
Ajay Badminton Green Nokia

I.Study the following information carefully to answer the given


questions

There are six boys M, N, O, P, Q and R. They want to go out with six girls S,
T, U, V, W and X, not necessarily in the same order. The pairs went to
different cities Agra, Ooty, Darjeeling, Shimla and two of them went to Goa.
They like different bikes, viz Karizma, Bullet, Passion and Discover. But
Karizma and Bullet are preferred by two pairs.

 M and P visit Goa but do not like either Karizma or Discover.


 R does not go out with W Both of them do not like Passion.
 V and U want to go to Agra and Darjeeling respectively. The persons
visiting Ooty like the same bike as N does.
 O goes out with X to Ooty but does not like either Bullet or Discover.
 N goes to Agra and likes Karizma.
 Q does not go out either with S or T, he does not go to Darjeeling,
he likes neither Passion nor Bullet.

1. Who among the following went to Agra ?


1.PS
2.WQ
3.OX
4.NV
5.None of these

Answer

Answer – 4.NV

2. If M goes with S then who among the following went goes


with P ?
1.V
2.U
3.T
4.W
5.None of these

Answer

Answer – 3.T

3. Who among the following went to ooty ?


1.X
2.U
3.V
4.W
5.None of these

Answer

Answer – 1.X

4. Which of the following bike does O like ?


1.Karizma
2.Discover
3.Bullet
4.Passion
5.None of these

Answer

Answer – 1.Karizma

5. Which of the following is correct ?


1.O-Goa-Passion
2.R-Agra-Discover
3.U-Ooty-Bullet
4.Q-Shimla-Discover
5.None of these

Answer

Answer – 4.Q-Shimla-Discover

Boy Girl Place Bike


M S/T Goa Bullet/Passion
P T/S Goa Passion/Bullet
R U Darjeeling Bullet
O X Ooty Karizma
N V Agra Karizma
Q W Shimla Discover
II.Study the following information carefully to answer the given
questions

There are five executives A, B, C, D and E. Each has a different hobby viz
Reading, Shopping, Singing, Walking and Swimming but not necessarily in
the same order. Each executive has friendship with a persons viz P, Q, R, S
and T. Each persons has a different profession viz Doctor, Lawyer, Teacher,
Engineer and Driver.

 Among Executives , there are only three females – the one who has
friendship with R, the one whose hobby is Reading and the one who
has friendship with the person who is an Engineer.
 The female whose hobby is shopping has friendship with the person
who is Teacher.
 The Person who is a lawyer has no friendship with the person whose
hobby is Singing. S is neither a lawyer nor a Doctor.T is not
Teacher.
 The person who has friendship with T is a female but she is not D.
E’s hobby is shopping and is not a male. A and C are not females.
 A’s hobby is swimming and is friend neither with T nor with Lawyer.
 B’s hobby is walking. Q is a Doctor and he has no friendship with A.
The one who is a Doctor has no friendship with D.

6. The one whose hobby is singing is a friend of who among the


following ?
1.R
2.Lawyer
3.T
4.Doctor
5.None of these

Answer

Answer – 4.Doctor

7. Which of the following pairs correctly represents the pair of


friends ?
1.CR
2.AS
3.DT
4.RB
5.None of these

Answer

Answer – 2.AS

8. Who among the following is an Engineer ?


1.T
2.S
3.R
4.P
5.None of these

Answer

Answer – 1.T

9. Which of the following represents the group of persons


having female friend ?
1.T,R, S
2.S,Q, R
3.R, P, T
4.T,S, Q
5.None of these

Answer

Answer – 3.R, P, T

10. Which of the following is the specialisation of S ?


1.Driver
2.Engineer
3.Teacher
4.Doctor
5.None of these
Answer

Answer – 1.Driver

Executive Hobby Friend Profession


E(-) Shopping R Teacher
D(-) Reading P Lawyer
B(-) Walking T Engineer
A(+) Swimming S Driver
C(+) Singing Q Doctor

I. Study the following information carefully to answer the given


questions
Eight People – A, B, C, D, E, F, G and H live in eight different floors of
building (but not necessarily in the same order). The lowermost floor of the
building is numbered one, the one above that is numbered two, and so on till
the topmost floor is numbered eight. Each one of them also owns a different
car, namely Santro, Brio, Amaze, Civic, Etios, Celerio, Micra and Fabia (but
not necessarily in the same order).

 F lives an odd numbered floor above the floor numbered four. Only
one person lives between F and the one who owns Amaze. The
number of people leaving above F is same as the number of people
living between F and D.
 Only three people live between D and the one who owns Celerio. C
lives an one of the odd numbered floors above the one who owns
Celerio.
 Only two people live between C and the one who owns Santro. The
one who owns Micra lives immediately above G, G owns neither
celerio nor Santro. E does not own Micra.
 Only three people live between G and A. The one who owns Fabia
lives immediately above the one who owns Brio, but not on the
topmost floor.
 Only one person lives between the one who owns Fabia and H. Only
one person lives between B and the one who owns Etios.

1. Which of the following Statements is true with respect to the


given information?
1. G lives immediately above the one who owns Celerio
2. E lives immediately above C
3. Only three people live between F and the one who owns Fabia.
4. D owns Etios
5. All the given statements are true.

Answer

Answer – 1. G lives immediately above the one who owns


Celerio

2. Who amongst the following lives exactly between H and the


one who owns Brio?
1. D, the one who owns Fabia
2. A, the one who owns Amaze
3. F, E
4. A, the one who owns Santro
5. E, D

Answer

Answer – 2. A, the one who owns Amaze

3. Four of the following cars does E own?


1. Civic
2. Celerio
3. Brio
4. Amaze
5. Fabia

Answer

Answer – 4. Amaze
4. Four of the following five are alike in a certain way and so
form a group. Which
one of the following does not belong to the group?
1. G – Civic
2. C – Celerio
3. F – Etios
4. H – Fabia
5. D – Amaze

Answer

Answer – 3. F – Etios

5. How many people live between C and the one who owns
Amaze?
1. Four
2. Three
3. Two
4. Five
5. None

Answer 2. Three

Floor No Person Car


8 B Civic
7 C Micra
6 G Etios
5 F Celerio
4 H Santro
3 E Amaze
2 A Fabia
1 D Brio

Seven people A, B, C, D, E, F and G were appointed to a company on seven


different days of the same week starting from Monday to Sunday(but
not necessarily in the same order). Each person also plays a different
game namely – Cricket, Hockey, Football, Squash, Volleyball, Tennis and
Kho-Kho.(but not necessarily in the same order).

 Only two people were appointed after the one who plays Hockey. E
was appointed on one of the days after the one who plays Hockey.
Only three people were appointed between E and G.
 Only one person was appointed between G and the one who plays
Volleyball. A was appointed immediately after the one who plays
volleyball. only three people were appointed after the one who plays
Kho-Kho.
 C was appointed immediately after F but not on Friday. Only one
people was appointed between F and the one who plays cricket. B
was appointed immediately after the one who plays cricket.
 More than two people were appointed between D and the one who
plays tennis. E does not play Football and Tennis

6. Who amongst the following was appointed on Wednesday?


1. The one who plays Kho-Kho
2. A
3. B
4. The one who plays Cricket
5. F

Answer

Answer – 4. The one who plays Cricket

7. Which of the following statements is true as per the given


arrangement?
1. Only one person was appointed between F and the one who plays
Squash
2. Only three people were appointed before C
3. A plays Tennis
4. B was appointed on Saturday
5. None of the given options is true

Answer
Answer – 1. Only one person was appointed between F and
the one who plays Squash

8. The person who plays Tennis was appointed on which of the


following days?
1. Thursday
2. Tuesday
3. Sunday
4. Monday
5. Saturday

Answer

Answer – 5. Saturday

9. Which of the following combinations will be definitely true as


per the given arrangement?
1. C – Squash
2. Thursday – D
3. Saturday – Cricket
4. Monday – Volleyball
5. F – Kho-kho

Answer

Answer – 4. Monday – Volleyball

10. How many people were appointed before G?


1. Two
2. One
3. Three
4. Four
5. None as G was appointed on Monday

Answer

Answer – 1. Two

Days Persons Sports


Monday D Volley Ball
Tuesday A Foot Ball
Wednesday G Cricket
Thursday B Kho – Kho
Friday F Hockey
Saturday C Tennis
Sunday E Squash

I. Study the following information carefully to answer the given


questions
Seven Persons – M, N, O, P, Q, R and S – live on separate floors of a seven
storeyed building, but not in the same order. The ground floor of the building
is numbered 1, the floor above it 2 and so on until the topmost floor is
numbered 7.Each person likes different cartoon characters, viz, Chipmnuk,
Flinstone, Jetson, Popeye, Scooby Doo, Simpson and Tweety, but not
necessarily in the same order.

 The person who likes Popeye lives on floor numbered 4. Only two
persons live between P and the one who likes Popeye. M does not
live on the lowermost floor. M lives on any odd numbered floor
below the one who likes Popeye.
 S lives on an even numbered floor but neither immediately above
nor immediately below the floor of M.Only two persons live between
M and the person who likes Tweety. Only one person lives between
N and R. R lives on an even numbered floor and does not like
Popeye.
 Only three persons live between the persons who like Chipmnuk and
Jetson respectively. The person who likes Chipmnuk live on any
floor above the N’s floor. The person who likes Chipmnuk does not
live on the topmost floor.
 O does not like Chipmnuk or Jetson. The person who likes Scooby
Doo lives on the floor immediately above the floor of the person who
likes Simpson.
1. How many persons live between the floors on which S and P
live?
A. Three
B. Two
C. Four
D. Five
E. No one

Answer

Answer – C. Four

2. Which of the following statements is/are true according to


the given information?
A. Q lives on floor numbered 5 and he does not like Popeye
B. M likes Scooby Doo and he does not live on floor numbered 4
C. O likes Flinstone and he lives on the topmost floor
D. Only two persons live between the floors of Q and R
E. All the statements are true.

Answer

Answer – E. All the statements are true.

3. Who among the following lives on the floor immediately


above the floor of M?
A. N
B. R
C. S
D. O
E. No one

Answer

Answer – A. N

4. Who among the following lives exactly between the floors on


which S and N live?
A. P
B. R
C. M
D. Q
E. No one

Answer

Answer – D. Q

5. Who among the following does like cartoon character Jetson?


A. R
B. P
C. N
D. Q
E. S

Answer

Answer – B. P

Floor No Person Cartoon Character


7 O Flinstone
6 S Tweety
5 Q Chipmnuk
4 N Popeye
3 M Scooby Doo
2 R Simpson
1 P Jetson

II. Study the following information carefully to answer the given


questions
Seven friends T, U, V, W, X, and Z are students of a school studying in
different subjects VI, VII and VIII. Not less than two and not more than three
students study in each standard each of them likes to play a different sport
in his spare time viz, Carrom, Badminton, Chess, Scrabble, Cards, Table
Tennis and Basket Ball, but not necessarily in the same order.
 X does not study in standard VI and likes Table Tennis.
 The one who like Chess studies in Standard VIII.
 U studies in Standard VII only with Y.
 V likes Carrom and studies in the same standard as W and T.
 No one studying in Standard VI like Badminton and Cards.
 W does not like Basket Ball and U does not like cards

6. Which Sport does U like?


A. Table Tennis
B. Badminton
C. Basket Ball
D. Carrom
E. Cannot be determined

Answer

Answer – B. Badminton

7. Who likes to play Scrabble?


A. W
B. U
C. T
D. Z
E. None of these

Answer

Answer – A. W

8. Which of the following represents the students studying in


Standard VIII?
A. X, Y
B. Z, T
C. X, Z
D. U, Y
E. None of these

Answer
Answer – C. X, Z

9. Which of the following combinations of Student – Standard –


Sport is correct?
A. W – VIII – Scrabble
B. U – VI – Badminton
C. T – VI – Cards
D. Z – VIII – Chess
E. None is correct

Answer

Answer – D. Z – VIII – Chess

10. Which Sport does Y like?


A. Scrabble
B. Cards
C. Badminton
D. Chess
E. Can not be determined

Answer

Answer – B. Cards

Students Class Games


U VII Badminton
Y VII Cards
X VIII Table Tennis
Z VIII Chess
V VI Carrom
W VI Scrabble
T VI Basket Ball

Study the following information carefully to answer the given


questions
There are seven friends namely A, B, C, D, E, F and G each of them aspires
to become something.
Each of them likes different colours viz. Blue, Yellow, Green, Violet, Red,
Black and Pink but not necessarily in the same order.
Each of them wants become Lawyer, Doctor, Engineer, Actor, Pilot, Soldier
and Cricketer but not necessarily in the same order.
E likes violet colour and wants to become a Doctor.
C likes yellow colour and wants to become a Cricketer.
G doesn’t like black colour and doesn’t want to become an Engineer.
The person who likes red colour, wants to become an Actor while the person
who wants to be a Pilot, likes blue colour.
A likes pink colour but F doesn’t like green or black colour and he doesn’t
want to become an Actor.
D wants to become a Lawyer. B doesn’t want to become an Engineer or a
soldier.

1. Who does want to become a soldier?


A. G
B. C
C. F
D. A
E. B

Answer

Answer – A. G

2. Who among the following likes red colour?


A. G
B. C
C. F
D. A
E. B

Answer

Answer – E. B
3. The person who wants to become a lawyer likes which
colour?
A. Red
B. Green
C. Black
D. Blue
E. Pink

Answer

Answer – C. Black

4. F wants to become a/an ________


A. Actor
B. Doctor
C. Lawyer
D. Pilot
E. Engineer

Answer

Answer – D. Pilot

5. Which of the following combinations of Person – Colour –


Profession is correct?
A. D – Blue – Lawyer
B. F – Blue – Pilot
C. A – Red – Cricketer
D. B – Green – Actor
E. G – Red – Actor

Answer

Answer – B. F – Blue – Pilot

Person Profession Colour


A Engineer Pink
B Actor Red
C Cricketer Yellow
D Lawyer Black
E Doctor Violet
F Pilot Blue
G Soldier Green

II. Study the following information carefully to answer the given


questions

There are five married couples in a family and there is a child to every
couple. Ages of children are 3,5,6,4 and 9 years.
Names of male are Rahul, Harish, Sourav, Ritesh and Mohit but not
necessarily in the same order.
Names of female are Geeta, Veena, Sita, Nita and Rachita. but not
necessarily in the same order.
Names of children are A, B, C, D & E. but not necessarily in the same order.
Name of Rahul’s child is not C and E and he is not eldest or youngest.
Age of Sita’s child is 6 years and her husband is one among Harish, Sourav
and Mohit.
Age of D is 3 years but she is not a child of Sourav.
A’s age is multiple of 3 but she is not a child of Rahul and Geeta.
Veena’s husband is Sourav or Mohit
Age of Rachita’s child is 5 years but the name of the child is not B and E.
Geeta is wife of Sourav.

6. What is the age of B?


A. 6
B. 4
C. 5
D. 9
E. Can’t be determined

Answer

Answer – B. 4
7. What is the name of Rahul’s wife?
A. Nita
B. Sita
C. Rachita
D. Veena
E. Can’t be determined

Answer

Answer – A. Nita

8. Name of father, mother and age of E?


A. Mohit, Nita & 6
B. Rahul, Sita & 9
C. Sourav, Geeta & 9
D. Ritesh, Veena & 6
E. Can’t be determined

Answer

Answer – C. Sourav, Geeta & 9

9. What is the name and the age of Ritesh’s child?


A. B & 6
B. E & 9
C. C & 4
D. C & 5
E. Can’t be determined

Answer

Answer – D. C & 5

10. What are the names of Mohit’s wife and his child?
A. Sita & A
B. Veena & D
C. Nita & D
D. Rachita & A
E. Can’t be determined
Answer

Answer – B. Veena & D

Husband Wife Children Age


Sourav Geeta E 9
Ritesh Rachita C 5
Mohit Veena D 3
Rahul Nita B 4
Harish Sita A 6

You might also like